87
PRESTORMING TM 2021 SET 2 - TEST 15 HISTORY III - EXPLANATION Q.1) If you travel from Delhi to Bhopal by train, you will see the majestic monument on top of a hill. If you climb up the hill you can see the complex of structures decorated with some of the best Buddhist artwork in the world. The initial structure was built from large bricks and mud mortar. The structure was enlarged using local sandstone during the Sunga period. The elaborately carved gateways were added later, in the 1st century BC. Which of the following monuments are you looking at? (a) Mahabodhi temple (b) Sanchi Stupa (c) Khajuraho temple complex (d) Bhimbetka caves EXPLANATION: Sanchi Stupa is the majestic monument on top of a hill at Sanchi Town in Raisen District of the State of Madhya Pradesh. So if one travels by train from Delhi to Bhopal he/she can witness it as it is located 46 kilometers north-east of Bhopal. It has a complex of structures decorated with some of the best Buddhist artwork in the world. It was initially built by the Mauryan emperor Ashoka in the 3rd century BCE with large bricks and mud mortar. Later the structure was enlarged to double its size using local sandstone covering the original dome during the Sunga period. The 4 elaborately carved gateways called ‗toranas‘ and an ornamented balustrade were also added to the main structure later, in the 1st century BC during the Satavahana rule. The architecture of the Sanchi Stupa includes harmika, chhatra, anda, vedica, etc. Anda is the hemispherical dome, harmika is a square railing, chhatra is an umbrella on top of the harmika. It has lower and upper pradakshinapatha for worshippers to be able to walk around the stupa. It is also a UNESCO world heritage site since 1989. Hence the monument that is being looked at is Sanchi Stupa. So Option (b) is correct. ADDITIONAL INFORMATION: Mahabodhi Temple: Located in Bodh Gaya, Bihar Situated on the banks of the Niranjana River Marks the location of Buddha‘s Enlightenment Originally built by the Mauryan emperor Ashoka

PRESTORMING 2021 SET 2 - TEST 15 HISTORY III - EXPLANATION

  • Upload
    others

  • View
    4

  • Download
    0

Embed Size (px)

Citation preview

Page 1: PRESTORMING 2021 SET 2 - TEST 15 HISTORY III - EXPLANATION

PRESTORMINGTM 2021

SET 2 - TEST 15 HISTORY III - EXPLANATION

Q.1) If you travel from Delhi to Bhopal by train, you will see the majestic monument on top of a

hill. If you climb up the hill you can see the complex of structures decorated with some of the

best Buddhist artwork in the world. The initial structure was built from large bricks and mud

mortar. The structure was enlarged using local sandstone during the Sunga period. The

elaborately carved gateways were added later, in the 1st century BC.

Which of the following monuments are you looking at?

(a) Mahabodhi temple

(b) Sanchi Stupa

(c) Khajuraho temple complex

(d) Bhimbetka caves

EXPLANATION:

Sanchi Stupa is the majestic monument on top of a hill at Sanchi Town in Raisen District of the State of

Madhya Pradesh. So if one travels by train from Delhi to Bhopal he/she can witness it as it is located 46

kilometers north-east of Bhopal. It has a complex of structures decorated with some of the best

Buddhist artwork in the world.

It was initially built by the Mauryan emperor Ashoka in the 3rd century BCE with large bricks and mud

mortar. Later the structure was enlarged to double its size using local sandstone covering the original

dome during the Sunga period. The 4 elaborately carved gateways called ‗toranas‘ and an ornamented

balustrade were also added to the main structure later, in the 1st century BC during the Satavahana

rule.

The architecture of the Sanchi Stupa includes harmika, chhatra, anda, vedica, etc. Anda is the

hemispherical dome, harmika is a square railing, chhatra is an umbrella on top of the harmika. It has

lower and upper pradakshinapatha for worshippers to be able to walk around the stupa. It is also a

UNESCO world heritage site since 1989.

Hence the monument that is being looked at is Sanchi Stupa. So Option (b) is correct.

ADDITIONAL INFORMATION:

Mahabodhi Temple:

Located in Bodh Gaya, Bihar

Situated on the banks of the Niranjana River

Marks the location of Buddha‘s Enlightenment

Originally built by the Mauryan emperor Ashoka

Page 2: PRESTORMING 2021 SET 2 - TEST 15 HISTORY III - EXPLANATION

Khajuraho Temple Complex:

Located in Madhya Pradesh

It is a UNESCO World Heritage Site

It was constructed from the period of 950 to 1050 AD by the rulers of the Chandela dynasty

Famous for erotic sculptures in these temples

Associated to two religions – Jainism and Hinduism

Kandariya Mahadeva Temple is the largest and the most famous

Nagara style of architecture

Bhimbetka Caves:

Located in the foothills of the Vindhyan Mountains on the southern edge of the central Indian

plateau in Madhya Pradesh

It is surrounded by the Ratapani Wildlife Sanctuary

It is known for prehistoric rock shelters and rock paintings belonging to the Mesolithic period

It was designated a UNESCO World Heritage site in 2003

The paintings found in the rock shelters here have a striking resemblance to the ones discovered in

Kakadu National Park in Australia; to the cave paintings of Bushmen in the Kalahari Desert and

Upper Palaeolithic Lascaux cave paintings in France

Q.2) The Prakrit text Uttaradhyayana Sutta belongs to

(a) Buddhism

(b) Jainism

(c) Aivika sect

(d) Lokayata philosophy

EXPLANATION:

Uttaradhyayana Sutta is a sacred text of the Svetambara sect of Jainism written in Prakrit. It describes

how a queen named Kamalavati tried to persuade her husband to renounce the world. It belongs to one

of the four 'Mulasutras' of the Jains. It is believed by orthodox Jains to contain the actual words of

Mahavira. It consists of 36 Chapters interspersed with lively narratives from folk literature and deals

with aspects of Jain doctrine and discipline. So Option (b) is correct.

ADDITIONAL INFORMATION:

Important books related to Jainism:

Tattvartha Sutra

Satkhandagama

Kalpa Sutra

Adi Purana

Page 3: PRESTORMING 2021 SET 2 - TEST 15 HISTORY III - EXPLANATION

Jnanarnava

Silappatikaram

Valayapathi

Q.3) A person visits a site and finds the following features

1. Yashti

2. Harmika

3. Chhatri

4. Anda

Which of the following place is the person visiting?

(a) Kandariya Mahadeo temple, Khajuraho.

(b) Five Rathas, Mahabalipuram.

(c) Stupa, Bharhut

(d) Yakshini, Didargunj.

EXPLANATION:

Yashti is the mast or central pillar that symbolizes the cosmic axis and surmounted by a Chhatri

Harmika a balcony-like structure or a square railing on top of the Anda represents the abode of

the gods

Chhatri or Chhatraveli is a triple umbrella on top of the Harmika, which represents the three

jewels of Buddhism – the Buddha, the Dharma, and the Sangha

Anda is the simple semi-circular mound of earth or hemispherical dome of the stupa

All these features belong to the Stupa Architecture. Hence the person is visiting the Stupa in Bharhut.

So Option (c) is correct.

ADDITIONAL INFORMATION:

Kandariya Mahadeo Temple, Khajuraho:

It is the tallest, the largest, and the most stunning in the Khajuraho complex

With about 870 spectacular sculptures, it is considered to be the spiritual abode of Lord Shiva

It is most popular for its ornate architecture, including statues of beautifully adorned women

The structure has a shikhara (spire) about 31 m high that depicts Mount Kailash

The main spire is surrounded by 84 miniature spires (Urushringas)

Five Rathas, Mahabalipuram:

Built during the Pallava Dynasty in the 7th Century

It is located near Chennai along the Coromandel Coast of the Bay of Bengal in Tamil Nadu

It is an example of Dravidian style architecture

Page 4: PRESTORMING 2021 SET 2 - TEST 15 HISTORY III - EXPLANATION

Chiseled in the shape of rathas or chariots out of a large block of stone or monolith of granite

typifies monolithic Indian rock-cut architecture

The five rathas are named as ‗Dharmaraja Ratha‘, ‗Bhima Ratha‘, ‗Arjuna Ratha‘, ‗Nakula Sahadeva

Ratha‘, and ‗Draupadi Ratha‘ after the five Pandava brothers and their common spouse Draupadi

from the great Indian epic ‗Mahabharata‘

Yakshini, Didarganj:

It is one of the finest sculpture from the Maurya Dynasty

It depicts a life-size female figure

It is 5‟4” tall and carved out of a single piece/monolithic polished stone, made from Chunar

sandstone

The fly whisk is held by the yakshini in the right hand and the left hand is broken

Now it is housed at the Patna Museum in Bihar

Q.4) It is a group of 24 Buddhist caves also known as Pandav Loni. Carved between the 1st

century BC and 3rd Century AD, it belongs to the Hinayana period. It was later influenced by

the Mahayana sect. The site of this cave also depicts an excellent system of water management

indicated through the presence of water tanks carved out of solid rocks.

The above lines defect which of the following rock-cut caves?

(a) Nasik caves

(b) Barbara Caves

(c) Nagarjuna caves

(d) Ajanta Caves

EXPLANATION:

The Nasik Caves is a group of 24 Buddhist caves also known as Pandav Leni. It was earlier called

Trirashmi Caves as it is located on Trirashmi hills in Maharashtra. The caves have images of both

Buddha and Bodhisattvas. They were primarily built during the reign of the Satavahana ruler Krishna. It

has inscriptions dating to the period of three kingdoms: Kshatrapas, Satavahanas, Abhirs.

Built gradually over an extended period between the 1st century BC and 3rd Century AD, it belongs to

the Hinayana period. Later it was also influenced by the Mahayana sect. These rock-cut temples initially

were started by Jains and later turned into a significant center of Buddhism.

Most of the Caves are Viharas (Buddhist Monastries) and some are Chaityas (Prayer hall with Stupa).

The caves were the meeting places for monks to pray and study Buddha or Buddhism. The site of this

cave also depicts an excellent system of water management indicated through the presence of water

tanks carved out of solid rocks.

Hence the given lines depict the Nasik Caves. So Option (a) is correct.

Page 5: PRESTORMING 2021 SET 2 - TEST 15 HISTORY III - EXPLANATION

ADDITIONAL INFORMATION:

Barabar Caves:

It is located in Bihar

It is an example of Mauryan rock-cut architecture

They were granted to the Ajivika Sect by Dasharatha, grandson of Ashoka

Four caves that form the Barabar caves are Lomas Rishi, Sudama, Karan Chaupar, and Vishwa

Zopri

Nagarjuni Caves:

It is located in Bihar

It was built around 232 BCE by Dasharatha

These caves were also devoted to the Ajivika Sect by Dasharatha

Three caves at Nagarjuni are Gopi, Vadithi ka Kubha and Vapiya ka Kubha

Ajanta Caves:

It is located in Aurangabad district of Maharashtra

It is a series of 30 rock-cut caves

All the caves are associated with Buddhism

They were constructed over some time between the 2nd Century BC to 6th Century AD

It has different types of sculpture, architecture, and paintings

It is a UNESCO World Heritage Site

Q.5) The traveler‟s book of travels is called Rihla, written in Arabic. He served as a qazi of Delhi

for several years. The ruler in India sent him as an envoy to Mongol . He travelled extensively in

Syria, Yemen, Sri Lanka , India, Sumatra and China.

Which of the following traveler and ruler is referred to in the above passage?

(a) Al –Biruni and Mahmud of Ghazni

(b) Ibn Battuta and Muhammad Bin Tughlaq

(c) Abdur Razzaq Samarqandi and Mahmud of Ghazni

(d) Francois Bernier and Muhammad BinTuglaq

EXPLANATION:

Ibn Battuta‘s book of travels, called Rihla, written in Arabic, provides extremely rich and interesting

details about the social and cultural life in the subcontinent in the fourteenth century. He was a

Moroccan traveler and known for his expertise in Islamic religious law or sharia. Before he set off for

India in 1332-33, he had made pilgrimage trips to Mecca and had already travelled extensively in Syria,

Iraq, Persia, Yemen, Oman, and a few trading ports on the coast of East Africa.

Muhammad bin Tughlaq, the Sultan of Delhi was impressed by his scholarship and appointed him the

Page 6: PRESTORMING 2021 SET 2 - TEST 15 HISTORY III - EXPLANATION

qazi or judge of Delhi. He remained in that position for several years, until he fell out of favour and was

thrown into prison. Once the misunderstanding between him and the Sultan was cleared, he was

restored to imperial service and was ordered in 1342 to proceed to China as the Sultan‘s envoy to the

Mongol ruler. Then he also travelled to other countries like Maldives, Sri Lanka, Sumatra, and China.

Hence the traveller and ruler referred to in the above passage are Ibn Battuta and Muhammad Bin

Tughlaq respectively. So Option (b) is correct.

ADDITIONAL INFORMATION:

Al –Biruni:

A Persian Scholar of the 11th century

He accompanied Mahmud of Ghazni during his raid on India

He wrote the book ‗Tahqiq-i-Hind/Kitab-ul-Hind’

Abdur Razzaq Samarqandi:

He was a Persian Scholar of the 15th century

He visited India during the rule of Deva Raya II of the Vijayanagara Empire

He wrote his travels in the ‗Matla-us-Sadain wa Majma-ul-Bahrain‘

Francois Bernier:

He was a French physician, philosopher, and traveller

He visited India from 1656-1668 during the reign of Shah Jahan

He played the role of the physician to Prince Dara Shikoh and later was attached to the court of

Aurangzeb

He wrote the book ‗Travels in the Mughal Empire‘

Q.6) Nastaliq refers to

(a) Wandering monks

(b) A style of Calligraphy

(c) Chronicle of Kings

(d) Impressive audience hall in Mughul Courts

EXPLANATION:

Nastaliq refers to a style of Calligraphy characterized by a tendency to slope downward from right to left

and used mainly for Persian poetical writings and in Urdu and Malay manuscripts. It is traditionally the

predominant style in Persian calligraphy. It was developed in Iran in the 14th and 15th centuries. So

Option (b) is correct.

Page 7: PRESTORMING 2021 SET 2 - TEST 15 HISTORY III - EXPLANATION

ADDITIONAL INFORMATION:

Different Arabic Calligraphic styles:

Taliq

Shikaste

Naskh

Thuluth

Reqa

Diwani

Q.7) Your friend likes to know the views of foreign chroniclers about the society and

administration during the Viayanagara Period. Which of the following Chronicler‟s excerpts

would you suggest reading?

1. Domingo Paes

2. Abdur Razzaq

3. Barbosa

4. Ibn Batuta

Select the correct answer using the codes given below:

(a) 1, 2 and 4 only

(b) 1 and 3 only

(c) 1, 2 and 3 only

(d) 1, 2, 3 and 4

EXPLANATION:

Domingo Paes was a Portugese traveller, merchant and writer. He gave detailed accounts of all

historic descriptions of ancient city Hampi which was governed by the Vijaynagara Empire under

Krishna Deva Raya

Abdur Razzaq was a Persian Scholar of the 15th century. He visited India during the rule of Deva

Raya II of the Vijayanagara Empire. He gives an account of the reign of Devaraya II in his ‗Matla-

us-Sadain wa Majma-ul-Bahrain‘

Barbosa was a Portuguese writer and traveller. Through his book ‗An Account of Countries

Bordering the Indian Ocean and their Inhabitants‘, he gave a detailed account of governance in

Vijayanagara under Krishna Deva Raya.

Ibn Batuta was a Moroccan traveller whose book Rihla contains accounts about the reign of

Haihara-I, founder of the Vijayanagar Empire

Hence all the options are correct. So Option (d) is correct.

Page 8: PRESTORMING 2021 SET 2 - TEST 15 HISTORY III - EXPLANATION

ADDITIONAL INFORMATION:

Other important foreign travellers who visited Vijayanagara:

Nicolo de Conti- Venetian

Athanasius Nikitin- Russian

Ludvico de Vorthema- Italian

Fernao Nuniz- Portuguese

Marco Polo- Venetian

Q.8) With reference to Mughul India, the term “jins-i-kamil” refers to

(a) Village headman

(b) Revenue earning crops

(c) Irrigation Canals

(d) Agricultural Labourers

EXPLANATION:

The term jins-i-kamil in Mughal India means perfect crops. They were cash crops that brought in more

revenue. Crops such as cotton and sugarcane were jins-i kamil par excellence. Cotton was grown over a

great swathe of territory spread over central India and the Deccan plateau, whereas Bengal was famous

for its sugar. Such cash crops would also include various sorts of oilseeds and lentils.

So Option (b) is correct.

ADDITIONAL INFORMATION:

Important terms in Mughal India:

Village headman - Muqaddam or Mandal

Irrigation Canals - Nahr, Nala

Agricultural Labourers – Majur

Scavengers - Halalkhoran

Sons of boatmen – Mallahzadas

Accountant of the panchayat - Patwari

Hereditary holdings - Miras or Watan

Q.9) Muhammad Husayn of Kashmir is one of the finest Calligraphers. He was honoured with the

title “Zarrin qalam”(golden pen) in recognition of the perfectly proportioned curvature ]of his

letters.

In which of the following ruler‘s court was he present?

(a) Qutb-uddin-aibak

Page 9: PRESTORMING 2021 SET 2 - TEST 15 HISTORY III - EXPLANATION

(b) Akbar

(c) Sher Shah

(d) Kanishka

EXPLANATION:

Muhammad Husayn was from Kashmir. He lived from 1575 to 1605. He is one of the finest calligraphers

at Akbar‟s court. He was honoured with the title ―Zarrin qalam‖(golden pen) in recognition of the

perfectly proportioned curvature of his letters. One of his folio in the Nastaliq style of calligraphy is

famous.

So Option (b) is correct.

Q.10) Who is credited for translating Patanjali‟s work on grammar into Arabic and works of Euclid

into Sanskrit?

(a) Nuniz

(b) Al –Biruni

(c) Ibn Battuta

(d) Abul Fazl

EXPLANATION:

Al-Biruni‘s expertise in several languages allowed him to compare languages and translate texts. He

translated several Sanskrit works, including Patanjali‘s work on grammar called ‗Mahabhasya‘ into

Arabic. For his Brahmana friends, he translated the works of Euclid (a Greek mathematician) into

Sanskrit. He was familiar with translations and adaptations of Sanskrit, Pali, and Prakrit texts into

Arabic – these ranged from fables to works on astronomy and medicine. He was well versed in several

languages like Syriac, Arabic, Persian, Hebrew, and Sanskrit

So Option (b) is correct.

ADDITIONAL INFORMATION:

Author Works

Fernao Nuniz Writings on the capital of the Vijayanagara

Empire

Ibn Battuta Rihla written in Arabic

Abul Fazl Ain-i-Akbari written in the Persian

language

Q.11) Techniques such as „stone on stone‟ and „pressure flaking‟ are related to which of the

following in reference to ancient Indian History?

(a) Deciphering an ancient script

Page 10: PRESTORMING 2021 SET 2 - TEST 15 HISTORY III - EXPLANATION

(b) Manufacturing of seals

(c) Manufacturing of stone tools

(d) Ancient methods of agriculture

EXPLANATION:

In the stone on stone method, a stone was taken in one hand and was hit with another stone to

make a particular shape. The stone which was made into a tool was called the core.

In the pressure flaking technique, the core stone was placed on a firm surface or the ground. The

stone used as a hammer was placed on a bone resting on the core stone to remove flakes from

the core stone. These flakes were shaped into tools.

Hence both these techniques refer to the manufacturing of stone tools. So Option (c) is correct.

ADDITIONAL INFORMATION:

Other stone tool manufacturing techniques:

Hammer and Anvil Technique

Bipolar Technique

Hard Hammer Percussion Technique

Soft Hammer Percussion Technique

Q.12) Consider the following statements

1. There is no trace of the urban way of life in Rig Veda

2. Horse, tiger and bull are mentioned in the Rig Veda

3. The Rig Vedas speak only about the Aryans

Which of the statements given above is/are correct?

(a) 1 only

(b) 2 and 3 only

(c) 2 only

(d) 1, 2 and 3

EXPLANATION:

There is no trace of the urban way of life in the Rig Veda. It is more of a rural way of life with no urban

centres. So Statement 1 is correct.

Horse (asva) occurs 215 times and bull (vrishabha) 170 times in Rig Veda. Tiger and rhinoceros, which

are tropical animals, are not mentioned in the Rig Veda. So Statement 2 is not correct.

The Rig Vedas speak about not only the Aryans but also about the non-Aryan people, whom the Aryans

encountered in India. When the Rig Vedic people moved into India they came into conflict with people

whom they referred to as Dasyus or Dasas. So Statement 3 is not correct.

Page 11: PRESTORMING 2021 SET 2 - TEST 15 HISTORY III - EXPLANATION

ADDITIONAL INFORMATION:

Rig Vedic Culture:

Various units of habitation and divisions such as the janas, vis, gana, grama and kula are referred

to in the Rig Veda

They strongly believed that prayers, sacrifices, and rituals could offer support in their mundane life

The god Indra is called Purandara, which means destroyer of settlements

The Rig Veda refers to Arya varna and Dasa varna

According to the Purusha Sukta of the Rig Veda, the various varnas emerged thus: Brahmanas

from the mouth, the Kshatriyas from the arms, the Vaisya from the thighs, and the Sudra from the

feet of Purusha, when he was sacrificed

Kinship was the basis of the social structure of Rig Vedic society

Women had a respectable position

Society was essentially patriarchal with a preference for male children and cattle

Archaeological evidence points to the development of agriculture among the Rig Vedic people

The assemblies called sabha, samiti, vidhata, and gana are mentioned in the Rig Veda

Q.13) Consider the following statements with reference to Hathigumpha Inscription

1. It consists of seventeen lines incised in Brahmi letters

2. It states that the Emperor Kharavela had a liberal religious spirit

3. It consists of details about Buddhism in that region

Which of the statements given above is/are correct?

(a) 1 and 3 only

(b) 1 and 2 only

(c) 2 only

(d) 1, 2 and 3

EXPLANATION:

Hathigumpha inscription consists of seventeen lines incised in deep cut Brahmi letters on the

overhanging brow of a natural cavern called Hathigumpha in the southern side of the Udayagiri hill near

Bhubaneswar in Orissa. So Statement 1 is correct.

The inscription states that the emperor Kharavela had a liberal religious spirit. Kharavela describes

himself as the worshiper of all religious orders, the restorer of shrines of all gods. So Statement 2 is

correct.

It consists of details about Jainism in that region and not Buddhism. So Statement 3 is not correct.

ADDITIONAL INFORMATION:

Hathigumpha Inscription:

Hathigumpha Inscription is claimed to be first discovered by A. Sterling in 1820

Page 12: PRESTORMING 2021 SET 2 - TEST 15 HISTORY III - EXPLANATION

The inscription is dated to be 2nd century BCE to 1st century CE

It is located on the twin hills of Khandagiri and Udaygiri

It faces straight towards the Rock Edicts of Ashoka at Dhauli

Kharavela selected that place to record both his political and religious activities

Hathigumpha Inscription is a piece of valuable epigraphic evidence to illuminate Nanda, Maurya,

Mahameghavahan dynasties; images of contemporary North and South India, and even story of

foreign invading Yavans

It has Temporal, Geographical, Religious, Socio-cultural, Linguistic, and Locality connotations

Q.14) Consider the following statements regarding the Kalabhras dynasty.

1. They ruled at the southern part of the country during the Sangam period.

2. They patronised both Buddhists and Jainists during their reign.

Which of the statements given above is/are correct?

(a) 1 only

(b) 2 only

(c) Both 1 and 2

(d) Neither 1 Nor 2

EXPLANATION:

Kalabhras ruled the southern part of the country after the Sangam period (300 BC to 300 AD) from

300 AD to 600 AD. So Statement 1 is not correct.

They patronised both Buddhists and Jainists during their reign. It was characterized by considerable

literary activity in Tamil and the ascendancy of Buddhism and Jainism during this period. So

Statement 2 is correct.

ADDITIONAL INFORMATION:

Kalabhras dynasty:

They established their kingdom after defeating the Cholas, Cheras, and Pandyas of the Sangam

period

The region of their rule is called Tamilakam

Their period of rule is referred to as the ‗Dark Ages‘ in the history of Tamilakam

Literary sources for this period are Periapuranam, Seevaka Chinthamani, Kundalakesi,

Yapernkalam, Tamil Navalar Charithai, etc.

A new script called Vattezhuththu developed during this period

Many works under Pathinen Keezhkanakku were composed

They were displaced around the 7th century by the revival of Pallava and Pandya power

Page 13: PRESTORMING 2021 SET 2 - TEST 15 HISTORY III - EXPLANATION

Q.15) Consider the following statements with reference to prehistoric period

1. It is located in a fertile plain, near the Bolan Pass

2. It is one of the earliest villages that we know about

3. The site has the remains of square or rectangular houses

The above statements are about which one of the following archeological sites?

(a) Koldihwa

(b) Gufkral

(c) Mehrgarh

(d) Shortugai

EXPLANATION:

Mehrgarh is a site located in a fertile plain, near the Bolan Pass, one of the most important routes into

Iran. It was one of the first places where people grew barley and wheat and reared sheep and goats. It is

one of the earliest known villages.

Archaeologists have found evidence of animal bones here on excavation. They have found remains of

houses here. Most houses were square or rectangular. They usually had four or more compartments,

some of which may have been used for storage. Several burial sites have also been found in Mehrgarh.

So Option (c) is correct.

ADDITIONAL INFORMATION:

Koldihwa:

It is a Neolithic site located in Belan valley, Uttar Pradesh

The site hosts the earliest evidence for rice cultivation

Gufkral:

It is a Neolithic site located in the Kashmir region

Both agriculture & domestication of animals was practiced here

Tools and weapons made of polished stone, as well as bone, were used

Shortugai:

It is a trade centre in Afghanistan

Harappan people imported semi-precious stones called lapis lazuli from here

Q.16) Consider the following statements with reference to Harappan cities

1. Very often walls of baked brick were built around each part of the cities

2. In general, the part to the east of the cities was larger but lower and called as citadel

3. Kalibangan and Lothal had fire altars

Which of the statements given above is/are correct?

(a) 1 and 2 only

(b) 1 and 3 only

Page 14: PRESTORMING 2021 SET 2 - TEST 15 HISTORY III - EXPLANATION

(c) 3 only

(d) 1, 2 and 3

EXPLANATION:

Very often walls of baked brick were built around each part of the cities. The bricks were laid in an

interlocking pattern and that made the walls strong. So Statement 1 is correct.

In general, the part to the east of the cities was larger but lower and called as the lower town. Citadel

was the west part of the city that was smaller but higher. So Statement 2 is not correct.

Kalibangan and Lothal had fire altars where sacrifices may have been performed. There were elaborate

storehouses in Mohenjadaro, Harappa, and Lothal. So Statement 3 is correct.

ADDITIONAL INFORMATION:

Important Harappan cities:

Mohenjadaro- Pakistan

Harappa - Pakistan

Kalibangan - Rajasthan

Dholavira - Gujarat

Lothal - Gujarat

Surkotada - Gujarat

Rakhigarhi - Haryana

Banawali - Haryana

Chanhudaro - Pakistan

Sutkagendor - Pakistan

Amri - Pakistan

Q.17) Consider the following

Who was famous for music during Mughal period?

(a) Baz Bahadur

(b) Faizi

(c) AbulFazl

(d) Rahim Khan-i-Khanan

EXPLANATION:

Baz Bahadur was the last Sultan of Malwa. His accomplishments were a mastery of music and poetry.

Because of Baz Bahadur‘s interest in music and poetry, Mandu (capital of Malwa) had become a

celebrated center for music. Once Malwa was occupied and absorbed by Mughal ruler Akbar, Baz

Bahadur became his fugitive and was enrolled as a Mughal mansabdar. Abul Fazl wrote about Baz

Bahadur – “he was the best person of his time in music and Hindi songs.”So Option (a) is correct.

Page 15: PRESTORMING 2021 SET 2 - TEST 15 HISTORY III - EXPLANATION

ADDITIONAL INFORMATION:

Faizi:

He was a Persian poet who joined Akbar‘s suite during the seize of Chittor in 1568

He was one of the Navratnas of Akbar

Akbar was impressed by the scholarly aptitude of Faizi and appointed him the tutor of princes

Salim, Murad and Daniyal

Lilavati, the celebrated Sanskrit work in Maths by Bhaskaracharya, was translated into Persian by

him

Abul Fazl:

He was the younger brother of Faizi

He was one of the Navratnas of Akbar

He was the author of Akbarnama, the official history of Akbar‘s reign in 3 volumes

Abdul Rahim Khan-i-Khanan:

He was one of the Navratnas of Akbar

He had an excellent command of the Sanskrit language

Rahim is known for his Hindi dohe (couplets) and his books on astrology

Q.18) A term Imadad-i-mash stands for in the Mughal order

(a) Poll tax

(b) Pension to army officers

(c) Cultivation tax

(d) Revenue free land assigned to learned and the needy

EXPLANATION:

There was a large class of religious divines and learned men who in return for their services were granted

tracts of land for their maintenance. Such grants were called milk or madad-i-maash. In Akbar‘s time,

such grants totalled about three percent of the jama. Later on, it was also made hereditary by Bahadur

Shah.

So Option (d) is correct.

ADDITIONAL INFORMATION:

Jizya:

It is a tax/poll tax imposed on non-muslims by Muslim rulers

In 1579 Akbar abolished it

Aurangzeb re-imposed it in 1679

Page 16: PRESTORMING 2021 SET 2 - TEST 15 HISTORY III - EXPLANATION

Q.19) Which of the following is wrongly matched?

(a) Aurang-Deport for manufactured Goods.

(b) Dadan-Advance commission

(c) Hundi-Bill of Exchange

(d) Dalal-Broker

EXPLANATION:

Aurang: It is the Persian term for warehouse or depot for manufactured Goods

Dadan: It is the system of employment of Inter-State migrant labour. No working hours are fixed

for these workers and they have to work on all the days in a week under extremely bad working

conditions

Hundi: It is a note recording a deposit made by a person. The amount deposited can be claimed

in another place by presenting the record of the deposit and hence a Bill of Exchange

Dalal: It means an agent working for a commission in return for a service and also called a broker

So Option (b) is correct.

Q.20) Which was the earliest text that tried to explain caste system according to one‟s abilities

and inclinations?

(a) Manusmriti

(b) Bhagavat Gita

(c) Ramayana

(d) Yajnavalkiya

EXPLANATION:

In line with the ancient Indian tradition, Manusmriti distinguished various human beings based on

their basic tendencies and inclinations. The element of desire, action, and knowledge was found among

persons varying according to the proportion of three gunas of Rajas, Tamas, and Sattvik in them.

Human beings were classified based on the preponderance of a particular guna. Thus in according to

Manusmriti, the Satvik guna preponderates in Brahmins, Rajas Guna dominates in Kshatriyas and Tamas

Guna dominates in the Vaishyas and Shudras. The Manu-smriti prescribes to Hindus their dharma—i.e.,

that set of obligations incumbent on each as a member of one of the four social classes/varnas and

engaged in one of the four stages of life called ashramas.

So Option (a) is correct.

ADDITIONAL INFORMATION:

Bhagavat Gita:

It is a 700 verse Hindu scripture in Sanskrit that is part of the Hindu epic Mahabharata

Page 17: PRESTORMING 2021 SET 2 - TEST 15 HISTORY III - EXPLANATION

It is set in a narrative framework of a dialogue between Pandava prince Arjuna and his guide and

charioteer Lord Krishna

The Bhagavad Gita presents a synthesis of the concept of Dharma, theistic bhakti, yogic ideals of

moksha through jnana, bhakti, karma, and Raja Yoga and Samkhya philosophy

Ramayana:

It is shorter of the two great epic poems of India, the other being the Mahabharata

It was composed in Sanskrit, probably not before 300 BCE, by the poet Valmiki and in its present

form consists of some 24,000 couplets divided into seven chapters

It is believed to be written in Treta Yuga

Ramayana describes the life of Rama, the eldest son of the king of Ayodhya, Dasaratha

Yajnavalkiya Smriti:

It is a book related to the Dharma in Hinduism

It was composed in the period of 3rd to 5th century CE in the Sanskrit language

It is organized into three books: Achara-kanda, Vyavahara-kanda, and Prayascitta-kanda

It consists of a cumulative total of 1,010 slokas (verses)

Q.21) Milk, Inam, Waqf were abolished by

(a) Iltutmish

(b) Balban

(c) AlauddinKhilji

(d) Guptas

EXPLANATION:

Milk, Inam, Waqf were land grants in the period of Delhi Sultanate. This type of land was given to the

people in gift or charity and particularly to Muslim scholars and saints. It was free of tax. Mostly the

land revenue was collected in cash but sometimes in kind also.

Milk, Inam, and Waqf were abolished by the Slave dynasty ruler Iltutmish. So Option (a) is correct.

ADDITIONAL INFORMATION:

Balban:

He Adopted the policy of ‗Blood and Iron‘ in governance

He introduced the Persian festival of Navroz in India

He ended the influence of the Corp of Forty

He introduced the practice of Sijda and Paibos

Alauddin Khilji:

He ruled for about twenty years from 1296 to 1316

He liked to call himself ‗The Second Alexander‘ and took the title ‗Sikander-i-Sani‘

Page 18: PRESTORMING 2021 SET 2 - TEST 15 HISTORY III - EXPLANATION

Malik Kafur was his powerful Army Chief

He Introduced Agrarian and Market reforms

He favoured centralised administration that was free from the influence of Ulemas

The system of espionage organised by him was systematic and effective

Q.22) The most distinctive characteristic in Muslim architecture that of Hindu and Buddhist

architecture was

(a) Round domes

(b) High minarets

(c) No concrete

(d) Combination of arch and dome

EXPLANATION:

The hemispherically shaped Stupas of the Buddhist architecture usually housed sacred relics associated

with the Buddha or other saintly persons. Domes in pre-Mughal India have a standard squat circular

shape with a lotus design and bulbous finial at the top, derived from Hindu architecture. Because the

Hindu architectural tradition did not include arches, flat corbels were used to transition from the

corners of the room to the dome, rather than squinches.

Hence the distinctive characteristic in Muslim architecture adopted from that of Hindu and Buddhist

architecture was Round domes. So Option (a) is correct.

ADDITIONAL INFORMATION:

Characteristics of Indo-Islamic Architecture:

Arches and Domes

Tall Minarets

Usage of mortar as cementing agents

Pietra dura technique was used

Usage of arabesque method

Presence of symmetry

Avoidance of representation of humans

Presence of water in premises

Q.23) Which of the following is correct regarding Alauddin Khilji?

(a) He abolished khuts and Muqqadams

(b) He was the first to collect taxes directly from the peasants

(c) He retained khuts and Muqqadams but reduced their privileges

(d) He appointed special officers in place of khuts and Muqqadams.

Page 19: PRESTORMING 2021 SET 2 - TEST 15 HISTORY III - EXPLANATION

EXPLANATION:

In the area brought under khalisa, Alauddin tried to curb the privileges of the khuts, muqaddams, and

chaudharis. Alauddin not only forced the khuts, muqaddams, and chaudharis to pay the grazing and

house taxes like the others, but also through the system of measurement ensured that they could not

pass on their burden of land revenue on the shoulders of the others. They were also deprived of the

khuti charges for collecting land revenue.

Hence Alauddin Khilji retained khuts and Muqqadams but reduced their privileges. So Option (c) is

correct.

ADDITIONAL INFORMATION:

Alauddin Khilji:

He ruled for about twenty years from 1296 to 1316

He liked to call himself ‗The Second Alexander‘ and took the title ‗Sikander-i-Sani‘

Malik Kafur was his powerful Army Chief

He Introduced Agrarian and Market reforms

He favoured centralised administration that was free from the influence of Ulemas

The system of espionage organised by him was systematic and effective

He established a proper postal system for establishing regular contact with various parts of the

empire

Q.24) Pahori painting emphasised on

(a) Snow-clad mountains

(b) Birds and Animals

(c) Love

(d) Seasons

EXPLANATION:

Pahari paintings emphasised areas like love and devotion. The artists adopted themes of eternal love

between Radha and Krishna. Gita Govinda (the Divine Love Song) and the tenth book of the Bhagavata

Purana (the stories of Krishna) provided evergreen themes. The growing popularity of vernacular

literature with the emergence of the Bhakti movement provided themes for the Pahari Paintings.

Erstwhile Shaiva- Shakta themes in paintings were later accompanied by the vernacular poetry and folk

songs adoring Krishna and Rama.

So option (c) is correct.

Page 20: PRESTORMING 2021 SET 2 - TEST 15 HISTORY III - EXPLANATION

ADDITIONAL INFORMATION:

Pahari Paintings:

Pahari denotes ‗hilly or mountainous‘ in origin

Pahari Schools of Painting includes towns, such as Basohli, Guler, Kangra, Kullu, Chamba,

Mankot, Nurpur, Mandi, Bilaspur, Jammu, and others in the hills of western Himalayas

These paintings established and developed during the period of 17th to 19th century

They have been done mostly in miniature painting forms

Based on geography two categories can be identified:

Basohli and Kulu style - shows the influence of the Chaurpanchasika style and emphasizes

the abstraction, bold lines, and conservative colors

Guler and Kangra style - underscores on calm, cooler colors and refinement

Pahari paintings have been widely influenced by the Rajput paintings, because of the family

relations of the Pahari Rajas with the royal court at Rajasthan

Unlike Rajasthani Paintings, which centered on portraitures, and depiction of splendid court life,

Pahari paintings emphasized love and devotional themes

The influence of the Mughals, Gujarat, and Deccan was also conspicuous in these Indian paintings

Q.25) Which of the following created a separate ministry for slaves?

(a) Itutmish

(b) AlauddinKhalji

(c) Balban

(d) Firoz Shah

EXPLANATION:

Firoz Shah Tughlaq increased the number of slaves by capturing the defeated soldiers and young

persons during the war. Hence he created a separate ministry or department for slaves called as Diwan-

i-Bandagan. The officer-in-charge of this department was the Wakil-i-dar.

So Option (d) is correct.

ADDITIONAL INFORMATION:

Department Created by

Diwan-i-Khairat (charity department) Firoz Shah Tughlaq

Diwan-i-mustakharaj (to realise arrears) Alauddin Khilji

Diwan- i-kohi (department of agriculture) Muhammad bin Tughluq

Page 21: PRESTORMING 2021 SET 2 - TEST 15 HISTORY III - EXPLANATION

Q.26) Which of the following is called yavanapriya?

(a) Pepper

(b) Silk

(c) Cotton

(d) Gold

EXPLANATION:

The term Yavanapriya is mentioned in ancient Sanskrit texts and Sangam literature. Yavanas meant the

Greeks, Romans, or in general foreigners from the west. Priya meant liking or love. Pepper was the most

liked and imported good by Greeks and other foreigners from India. Hence it is referred to as

Yavanapriya.

So Option (a) is correct.

ADDITIONAL INFORMATION:

Major exports in Ancient India:

Spices, wheat, sugar, indigo, sesame oil, cotton, parrot, live animals, and animal products –hides, skin,

fur, horns, tortoiseshell, and precious stones like pearls, sapphires, quartz, and turquoise, etc.

Major imports in Ancient India:

Horses, Chinese silk, flax and linen, wine, gold, silver, tin, lead, rubies, glass, and amber, etc.

Q.27) Which of the following is not a vedic text?

(a) Samhita

(b) Brahmana

(c) Dammadada

(d) Aranyakas

EXPLANATION:

Samhita, Brahmana, Aranyakas and Upanishads are Vedic texts.

Dhammapada is a Buddhist Pali text. It is the second text in the Khuddaka Nikaya of the Sutta Pitaka. It

contains a collection of the sayings of the Buddha in verse form.

So Option (c) is correct.

Page 22: PRESTORMING 2021 SET 2 - TEST 15 HISTORY III - EXPLANATION

ADDITIONAL INFORMATION:

Two types of Vedic literature:

Shruti Smriti

It means to hear It means to be remembered

It is canonical, consisting of revelation and

unquestionable truth, and is considered

eternal

It is supplementary and may change over time

It comprises of Vedas/Samhitas,

Brahmana, Aranyakas and Upanishads

It consists of Vedanga, Shad darsana, Puranas, Itihasa,

Upveda, Tantras, Agamas, Upangas

Q.28) Which among the following ruling dynasties has not been mentioned in the Sangam

literature:

(a) Kadamba

(b) Chera

(c) Chola

(d) Pandya

EXPLANATION:

In Sangam literature, there is mention of Cheras, Cholas, and Pandyas who ruled Southern India. The

Sangam period belongs to 300 BC to 300 AD.

Whereas the Kadambas were present in the period of 345 to 525 AD during the Gupta rule. It was a

primeval majestic dynasty of Karnataka that ruled from Vaijayanti or Banavasi in the present day Uttara

Kannada district.

Hence there is no mention of Kadambas in Sangam literature. So Option (a) is correct.

ADDITIONAL INFORMATION:

Dynasty Region Emblem Port

Cheras Central and northern parts of

Kerala and the Kongu region of

Tamil Nadu

Bow and arrow Tondi, Muziris

Cholas Kaveri Delta region called as

Cholamandalam

Tiger Puhar

Pandyas Madurai Fish Korkai

Page 23: PRESTORMING 2021 SET 2 - TEST 15 HISTORY III - EXPLANATION

Q.29) The most important feature concerning the land settlement during the Gupta-age was

(a) Increased land-revenue

(b) Collection of revenue in kind

(c) Beginning of feudalism or jagirdari system

(d) Collection of revenue through contractors

EXPLANATION:

Feudal development surfaced under the Guptas with the grant of fiscal and administrative concessions

to priests and administrators. Started in the Deccan by the Satavahanas, the practice became a regular

affair in Gupta times. Religious functionaries were granted land, free of tax, forever, and they were

authorised to collect from the peasants all the taxes which could have otherwise gone to the emperor.

Religious grants were of two types: Agrahara grants were meant for the Brahmanas which meant to be

perpetual, hereditary, and tax-free, accompanied with the assignment of all land revenue. The

Devagrahara grants were made to secular parties such as writers and merchants, for repair and worship

of temples.

Hence the most important feature concerning the land settlement during the Gupta age was the

collection of revenue through contractors. So Option (d) is correct.

ADDITIONAL INFORMATION:

Guptas:

Founded in the 3rd century AD by Srigupta

It is called the „Golden Age of India‟ because of the many achievements in the field of arts,

science, and literature

The king adopted the titles of such as Paramveshvara, Maharajadhiraja, and Parambhattaraka

Forced labour or Vishti was also practised in the royal army

The royal seal bore the imprint of Garuda

Famous works of literature during this period are Abhijnanashaakuntalam,

Kumarasambhavam, Malavikagnimitram, Mṛicchakatika, Panchatantra, Mudrarakshasa, etc.

Many magnificent temples, palaces, paintings, and sculptures were created during this period

Q.30) The main feature of the agrarian structure of the post-Gupta age was

(a) Direct relations of the state with peasants

(b) Jagirdari system

(c) Collection of revenue through village Panchayats

(d) None of the above

EXPLANATION:

The main feature of the agrarian structure of the post-Gupta age was jagirdari system which involved

the system of a land grant in return for services rendered. The widespread land-grant system during the

Page 24: PRESTORMING 2021 SET 2 - TEST 15 HISTORY III - EXPLANATION

post-Gupta period had created a separate group of landlords in the rural areas. From the seventh

century onwards the donee was given pastures, trees, water resources, etc. in addition to agricultural

land.

So Option (b) is correct.

Q.31) Which among the following ruling dynasties, successfully obstructed the efforts of the

Pratiharas in establishing a strong empire in North India?

(a) The Palas

(b) The Chalukyasof Badami

(c) The Rashtrakutas

(d) The Senas

EXPLANATION:

There were Tripartite struggles in Early Medieval North India. The three political powers during the

period were Pratiharas, Palas, and Rashtrakutas. All these kingdoms were in constant conflict with each

other and tried to gain control over the Gangetic region in north India and this struggle between the

three kingdoms is referred to as the Tripartite struggle. In these tripartite struggles Pratiharas

though they won a few battles, were defeated in many battles by the Rashtrakutas who

successfully obstructed the efforts of the Pratiharas in establishing a strong empire in North India.

So Option (c) is correct.

ADDITIONAL INFORMATION:

Pratiharas:

They are also called Gurjara- Pratihars because they originated from Gurjarata or south-western

Rajasthan

The kingdom was founded by Harichandra in Jodhpur (southwestern Rajasthan)

They ruled over western India and parts of northern India from the mid-7th century to the 11th

century

Significant rulers of the dynasty were Nagabhatta I, Nagabhata II, Mihir Bhoja, Mahenedrapala

They used the most common sandstones for idols that have various shades of red, caused by iron

oxide

Their style of architecture is known for their sculptures, carved panels, and open pavilion-style

temples belonging to Nagara Style of temple Architecture

The great Sanskrit poet and dramatist, Rajashekhar, lived at the court of Mahipala, grandson of

Bhoja

Q.32) The British started economic exploitation of the Indians when:

(a) There remained no fear of the Mughal emperor

Page 25: PRESTORMING 2021 SET 2 - TEST 15 HISTORY III - EXPLANATION

(b) There remained no fear of the might of the Marathas after the 3rd battle of Panipat

(c) No European rival after the 3rd Anglo-French was

(d) They captured political power in the provinces of Bengal, Bihar and Orissa after the battle of

Plassey.

EXPLANATION:

After the victory in the Battle of Plassey in 1757 and the Battle of Buxar in 1764, the whole dynamics of

the company‘s/British trade with India got changed. From then the company/British did not have to

bring any bullion for trade as it received the right to collect revenue/ Diwani right over Bengal, Bihar,

and Orissa through the Treaty of Allahabad of 1765.

Hence the British started economic exploitation of the Indians when they captured political power in the

provinces of Bengal, Bihar, and Orissa after the battle of Plassey. So Option (d) is correct.

ADDITIONAL INFORMATION:

3rd Battle of Panipat:

It was fought in the year 1761

The battle was fought between the Afghan ruler Ahmad Shah Abdali and Maratha forces

commanded by Sadashiv Rao Bhau & Vishwas Rao

It was won by Ahmad Shah Abdali

3rd Anglo-French war:

It is also called as 3rd Carnatic War

It was fought between 1758 to 1763

The reason for the war is the Seven years of war (17561763) in America

In 1760 Eyre Coot defeated Count de Lally at Wandiwash in 1760

The British emerged victoriously and it ended with the Treaty of Paris in 1763

Q.33) Which among the following did not emerge as a new social class during the British rule in

agrarian areas?

(a) Zamindars

(b) Absentee landlords

(c) Tenants under Zamindars and absentee landlords

(d) Bourgeoise farm holders

ADDITIONAL INFORMATION:

Commercialisation of agriculture by British:

It refers to change from cultivation for home consumption to cultivation for market consumption

Page 26: PRESTORMING 2021 SET 2 - TEST 15 HISTORY III - EXPLANATION

As the rate of revenue was high, peasants started producing such crops which had a high demand

in the market

Thus it lead to the cultivation of cash crops like cotton, jute, sugarcane, etc.

The Industrial Revolution in Britain lead to the increase in demand for raw materials

The British followed such policies which lead to the extension of areas cultivating cash crops

The Industrial Revolution accelerated the process of commercialisation and specialisation of Indian

agriculture

As villagers started producing for the market they were exposed to the erratic behaviour of the

market.

Q.34) Untouchables, a category of people lower than the Sudras came tobe recognised during the

Pre-Mauryan age primarily because of the development of:

(a) Agriculture

(b) Trade

(c) Handicrafts

(d) Their economic occupation

EXPLANATION:

Untouchables emerged as the fifth varna in the lowest level, which was considered a category of people

lower than the Shudras. It was the emergence of contempt for manual labour in the post-vedic period

that accounts for untouchability attached to groups who were deprived of any other means of

subsistence and engaged in manual labour to survive. It was also the impurity of certain occupations

assigned to particular castes and objects that led to untouchability like handling of meat, waste, dead

bodies, etc.

Hence Untouchables, a category of people lower than the Sudras came to be recognised during the Pre-

Mauryan age primarily because of the development of their economic occupation. So option (d) is

correct.

Page 27: PRESTORMING 2021 SET 2 - TEST 15 HISTORY III - EXPLANATION

ADDITIONAL INFORMATION:

Four Varnas and Untouchables:

Q.35) The philosophy of Upanishads emphasises on:

(a) Bhakti

(b) Gyan

(c) Karma

(d) Tapa

EXPLANATION:

Upanishad is derived from the words upa (near) and shad (to sit). It is derived from the concept of sitting

near the feet of the teacher. The word Upanishad means to sit down near someone and denotes a student

sitting near his guru to learn. Eventually, the word began to be used for the secret knowledge imparted

by the guru to his selected pupils. Upanishads specify philosophical knowledge and spiritual learning.

The main motto of the Upanishads is “Knowledge Awards Salvation”.

Hence the philosophy of Upanishads emphasises Gyan or knowledge. So Option (b) is correct.

ADDITIONAL INFORMATION:

Important Upanishads:

Aitareya Upanishad: Talks about the creation of Atman (Soul) and Consciousness

Page 28: PRESTORMING 2021 SET 2 - TEST 15 HISTORY III - EXPLANATION

Brihadaranyaka Upanishad: Talks about the transmigration of Atman; passages on metaphysics

and ethics

Chandogya Upanishad: Deals with rhythm and chanting of Mantras

Katho Upanishad: Tells the story of Nachiketa and Yama

Mundaka Upanishad: Contains the mantra ―Satyameva Jayate‖ (truth alone triumphs) which is

borrowed in the National Emblem of India

Svetasvatara Upanishad: In this Upanishad Siva or Rudra is declared to be the creator, preserver,

and destroyer of the world. He is the material and efficient cause of this world. He is identified with

the Supreme Brahman

Q.36) The largest concentration of Harappan sites has been found along the

(a) Sutlej

(b) Indus

(c) Ghaggar-Hakra

(d) Ravi

EXPLANATION:

The Ghaggar-Hakra river system is a seasonal river that flows through North-western India and

Pakistan. It is a rain-fed river and flows only during the monsoon season. The Ghaggar river originates

from the Siwalik Hills of Himachal Pradesh and flows through Punjab, Haryana, and then into

Rajasthan. The river is known as Ghaggar before the Ottu barrage and as the Hakra downstream of the

barrage.

The occurrence of a large number of Harappan settlements along the banks of the Ghaggar-Hakra

stream, which had remained a seasonal river for most of its historical existence, has baffled

archaeologists since the 1950s. Since the availability of water is the key to the development of a stable

urban civilization, many believe that the Ghaggar-Hakra river had a strong fluvial presence during the

Harappan times. The Ghaggar-Hakra river system also shares an identical geographic position with the

legendary glacier-fed river Saraswati mentioned in some of the ancient Indian scriptures. A few

researchers have also concluded the Ghaggar river is the Saraswati of Rig Vedic times.

So Option (c) is correct.

ADDITIONAL INFORMATION:

Harappan site River associated with the site

Harappa Ravi

Mohenjodaro Indus

Sutkagendor Dast

Page 29: PRESTORMING 2021 SET 2 - TEST 15 HISTORY III - EXPLANATION

Chanhudaro Indus

Amri Indus

Kalibangan Ghaggar

Lothal Bhogva

Q.37) Which of the following Harappan crops are believed to have diffused from West Asia?

1. Wheat

2. Rice

3. Cotton

4. Barley

Choose the correct answer from the codes given below:

(a) 1 and 4

(b) 2 and 3

(c) 1, 3 and 4

(d) 1, 2 and 4

EXPLANATION:

The crops and cattle in the Indus valley/Harappa were diffused mainly from Southwest Asia, though

there had been some native crops like legumes and sugarcane. Triticeae which is a clan of wild grass

included wheat, barley, rye, their wild relatives, and several important wild grasses. The Fertile Crescent,

at the core of western Asia and northern Africa, is the center of origin and early diversification of this

clan. Cotton was diffused here around 3000 BC. Ragi and bajra, indigenous to Africa, were cultivated in

south India in about 1500 BC. These crops, most probably arrived in India from Africa through the land

route of Southwest Asia.

The earliest evidence for rice across the world was found in Koldihwa located in Belan valley in Uttar

Pradesh. So it did not diffuse from West Asia.

Hence only 1, 3, and 4 are correct. So Option (c) is correct.

ADDITIONAL INFORMATION:

Crops grown in Harappa:

Wheat

Barley

Rice

Cotton

Peas

Sesame

Page 30: PRESTORMING 2021 SET 2 - TEST 15 HISTORY III - EXPLANATION

Linseed

Mustard

Chickpea

Millets

Q.38) Match List I with List II and select the correct answer by using the Codes given below the

lists:

List I

(Items found

in Harappan

sites)

List II

(Possible

Sources )

A. Copper 1. Deccan

B. Shell 2. Rajastan

C. Lapis Lazuli 3. Kutch

D. Gold 4. Afghanistan

Codes:

A B C D

(a) 3 4 1 2

(b) 1 3 2 4

(c) 2 3 1 4

(d) 2 3 4 1

EXPLANATION:

Copper as a raw material was procured by sending expeditions to the Khetri region of

Rajasthan. There is evidence in the Khetri area for what archaeologists call the Ganeshwar-

Jodhpura culture, with its distinctive non-Harappan pottery and an unusual wealth of

copper objects. It is possible that the inhabitants of this region supplied copper to the

Harappans.

Harappan settlements of Nageshwar and Balakot were specialised centres for making shell

objects – including bangles, ladles, and inlay – which were taken to other settlements.

Nageshwar and Balakot were located along the Kutch coast.

Lapis lazuli was a highly valued blue coloured stone that was procured or imported from

Shortughai in Afghanistan.

Gold found in Harappan sites was obtained by sending expeditions to the Deccan regions of

Southern India

Page 31: PRESTORMING 2021 SET 2 - TEST 15 HISTORY III - EXPLANATION

Hence the correct match is A-2, B-3, C-4, D-1. So Option (d) is correct.

ADDITIONAL INFORMATION:

Item found in Harappan site Possible source

Carnelian Bharuch in Gujarat

Steatite south Rajasthan and north Gujarat

Metal Rajasthan

Copper Rajasthan and Oman

Nickel Mesopotamian sites

Tin Afghanistan

Q.39) The doctrine of void (shunyata) was first expounded by the Buddhist philosopher

(a) Nagarjuna

(b) Nagasena

(c) Ananda

(d) Ashvaghosha

EXPLANATION:

Shunyata, in Buddhist philosophy, means the voidness that constitutes ultimate reality; Shunyata is

seen not as a negation of existence but rather as the un differentiation out of which all apparent entities,

distinctions, and dualities arise. Although the concept is encountered occasionally in early Pali texts, its

full implications were developed by the 2nd-century Indian philosopher Nagarjuna. The school of

philosophy founded by him, the Madhyamika (Middle Way), is sometimes called the Shunyavada, or

Doctrine That All Is Void.So Option (a) is correct.

ADDITIONAL INFORMATION:

Buddhist Scholar Famous work

Nagarjuna Mulamadhyamakakarika in Sanskrit

Nagasena Milinda Panha

Ashvaghosha Buddhacharita in Sanskrit

Vasubandhu Abhidharmamoksha

Buddhaghosa Visuddhimagga

Page 32: PRESTORMING 2021 SET 2 - TEST 15 HISTORY III - EXPLANATION

Chandrakirti Prasannapada

Q.40) Which of the following is/are true regarding Buddhism?

1. It did not reject varna and jati

2. It challenged highest social ranking of Brahmana varna

3. It regarded certain crafts as low

Choose the correct answer from the codes given below:

(a) 1 and 2

(b) 2 and 3

(c) 1, 2 and 3

(d) None of these

EXPLANATION:

Buddhism did not reject varna and jati but at the same time, they also did not promote the varna

system. Ancient Buddhist texts mention the Varna system in South Asia, but the details suggest that it

was non-rigid, flexible, and with characteristics devoid of features of a social stratification system.

According to this system, a person's position in society was determined from the time he was born and

there was no way to change his lot in life. In the initial stage, it did not attach any importance to the

existing Varna system. They preached the gospel of non-violence, which would put an end to wars

between different kingdoms and consequently promote trade and commerce. So Statement 1 is correct.

It challenged the highest social ranking of Brahmana varna. This is so because the Brahmanical law

books, called the Dharmasutras, decried lending money on interest. A person who lived on interest was

condemned by them. Therefore the Vaisyas, who lent money on account of growing trade and commerce,

were not held in esteem and were eager to improve their social status. Vaisyas were ranked third in the

Brahmanical society of the Vedic period and they looked for some religion that would improve their

position. The rituals and practices of Brahmanas were also seen to be complex, expensive, and

discriminatory. To overcome these problems they challenged Brahmanas. So statement 2 is correct.

Buddhism did not regard any craft as low. It treated all jobs/occupations done by the people equally. So

Statement 3 is not correct.

ADDITIONAL INFORMATION:

Four Noble truths of Buddhism:

1. The world is full of sorrows/sufferings

2. Desire is the root cause of sorrow/sufferings

3. If the desire is conquered all sorrows/sufferings can be removed

4. Desire can be removed by following the eight-fold path

Page 33: PRESTORMING 2021 SET 2 - TEST 15 HISTORY III - EXPLANATION

Q.41) Consider the following statements:

1. Harappans used both canal and well irrigation

2. Double cropping system was not known to Harappans

3. The Harappan pottery is well baked and fine in decorations

Which of the statements given above is/are correct?

(a) 1 and 3 only

(b) 2 and 3 only

(c) 1 and 2 only

(d) 1, 2 and 3

EXPLANATION:

Harappans used both canal and well irrigation. Most Harappan sites are located in semi-arid lands,

where irrigation was probably required for agriculture. Traces of canals have been found at the

Harappan site of Shortughai in Afghanistan. It is also likely that water drawn from wells was used for

irrigation. In Kalibangan many houses had their wells. So Statement 1 is correct.

Archaeologists have also found evidence of a ploughed field at Kalibangan in Rajasthan, associated with

Early Harappan levels. The field had two sets of furrows at right angles to each other, suggesting that

two different crops were grown together. Hence double cropping system was known to Harappans. So

Statement 2 is not correct.

The Harappan pottery is bright or dark red and uniformly sturdy and well baked. It consists chiefly of

the wheel-made wares both plain and painted. Plain pottery is more common than the painted ware. The

plain ware is usually of red clay with or without a fine red slip. The painted pottery is of red and black

colours. Several methods were used by people for the decoration of pottery. Geometrical patterns, circles,

squares, and triangles, and figures of animals, birds, snakes or fish are frequent motifs found in

Harappan pottery. Another favourite motive was the tree pattern. Plants, trees, and pipal leaves are

found on pottery. Harappan people used different types of pottery such as glazed, polychrome, incised,

perforated, and knobbed. So Statement 3 is correct.

ADDITIONAL INFORMATION:

Harappan/Indus Valley Civilisation:

Importanat urban centres of the civilisation are Harappa and Mohenjodaro in Pakistan, Lothal in

India, etc.

It had well-planned towns in a grid structure

The drainage system employed was impressive

Every big or small house had its courtyard and bathroom

There was the usage of burnt bricks for buildings

Page 34: PRESTORMING 2021 SET 2 - TEST 15 HISTORY III - EXPLANATION

Metal money was not used and trade was carried by barter system

The Harappans practised boat-making, bead making, and seal-making

Terracotta manufacture was also an important craft

Q.42) The term Nirgrantha is associated with

(a) Ajivikas

(b) Charvakas

(c) Jainas

(d) Pasupatas

EXPLANATION:

A Nirgrantha is a term that specifically refers to Jains in religious and philosophical works from the

Indian religions. It means they are people who are free from fetters or without any bonds. It was a title

given to ascetics who freed themselves from the knots which mean the household life of desires, duties,

and worries. Mahavira is often referred to as Nirgrantha in Buddhist texts.So Option (c) is correct.

ADDITIONAL INFORMATION:

Ajivika Sect:

The important leader of the sect was Makkhali Goshala

Their central belief was that absolutely everything is predetermined by fate or niyati, and hence

human action has no consequence one way or the other

The sect reached its peak during the Mauryan Emperor Ashoka‘s father‘s (Bindusara‘s) time

They were donated Barabar caves

Charvaka Philosophy:

It is also called Lokayata philosophy/philosophy of masses

Brihaspati is considered to be its founder

It is a philosophical Indian school of materialists who rejected the notion of an after world, karma,

liberation/moksha, the authority of the sacred scriptures, the Vedas, and the immortality of the

self

Its followers are referred to as hedonists advocating a policy of total opportunism

Pashupatas:

It was the earliest Hindu sect to worship the god Shiva as the supreme deity

It gave rise in turn to numerous subsects that flourished in Gujarat and Rajasthan, at least until

the 12th century

The Pashupata sect is mentioned in the Mahabharata

Page 35: PRESTORMING 2021 SET 2 - TEST 15 HISTORY III - EXPLANATION

Its doctrine was dualistic in character. Pasu the individual soul was eternally existing with Pati, the

supreme soul, and the attainment of Danhkhanta (cessation of misery) by the former was through

the performance of Yoga and Vidhi

Q.43) Which one of the following kingdoms was known for wealthy mechants professing Jainism

and building Jaina temples?

(a) The Gupta-Pratihara

(b) The Chaulukya

(c) The Gahadavala

(d) The Kakatiya

EXPLANATION:

Chalukyas ruled at Karnataka in the 6th to 8 century AD from Badami. Another branch of Chalukyas

ruled between 973 AD to 1186 AD from Kalyan on Karnataka, Maharashtra, and Andhra Pradesh. Many

kings, Queens, and Generals in both the branches were followers of Jainism. King Vinayaditya of

Badami was a pupil of Jain monk Udaidev Pandit. Ravikirti, General of Pulkeshin II was a Jain scholar

and Sanskrit pandit. The Jain temple at Meguti which is part of the Aihole temple complex was built

during the rule of early Chalukyas. The kings of the Chalukya dynasty, which ruled Gujarat and much

of Rajasthan and Malwa from the 10th to the late 13th centuries, were among the most generous

patrons, building numerous temples and libraries for Jainism. The Dilwara Temple complex consisting of

five ornately carved marble temples, each dedicated to a different Tirthankara was built under the

Chalukya rule.

Hence Chalukyas were known for wealthy mechants professing Jainism and building Jaina temples. So

Option (b) is correct.

ADDITIONAL INFORMATION:

Gahadavala Dynasty:

They ruled North India in the mid of 11th century till the mid of 13th century AD

Yashovigraha was the founder of the dynasty

The origin of this family is Banaras/Varanasi and Oudh/Ayodhya in Uttar Pradesh

Later they settled in Kannauj which became the Political centre of India

Important rulers of the dynasty were Yashovigraha, Mahichandra, and Chandradeva

Kakatiya Dynasty:

It is an Andhra dynasty that flourished in the 12th century

The Kakatiya dynasty ruled from Warangal (Telangana) from CE 1083-1323

They were known for the construction of a network of tanks for irrigation and drinking water

There are hundreds of Hindu temples built under the patronage of Kakatiya kings like Ganapati

Deva, Rudrama Devi, and Prataparudra

Page 36: PRESTORMING 2021 SET 2 - TEST 15 HISTORY III - EXPLANATION

Golconda Fort in Hyderabad was also constructed by the Kakatiya rulers

The Kakatiyas encouraged literature, art, and architecture

The thousand Pillar temple at Hanumakonda was built during their period and stands as an

everlasting contribution

Q.44) The Besnagar inscription of Heliodorus refers to

(a) Samkarshana and Vasudeva

(b) Samkarshana, Pradyumna and Vasudeva

(c) Vasudeva only

(d) All the Panchaviras

EXPLANATION:

The Heliodorus pillar is a stone column that was erected around 110 BCE in central India in Vidisha

near modern Besnagar, by Heliodorus, a Greek ambassador of the Indo-Greek king Antialcidas to the

court of the Sunga king Bhagabhadra. The pillar was surmounted by a sculpture of Garuda and was

dedicated by Heliodorus in honour of Vasudeva, the God of gods. Heliodorus described himself as a

Bhagavata and was a resident of Taxila.

Hence the Besnagar inscription of Heliodorus refers to Vasudeva only. So Option (c) is correct.

ADDITIONAL INFORMATION:

The 5 Vrishni heroes or Pancha-viras:

Samkarshana or Baladeva

Vasudeva-Krishna

Pradyumna

Samba

Aniruddha

Q.45) Which of the following terms denote monastic establishments of different religious

affiliations?

1. Vihara

2. Matha

3. Basadi

4. Mandapa

Choose the correct answer form the codes given below:

(a) 1 and 2 only

(b) 2 and 3 only

(c) 3 and 4 only

(d) 1, 2 and 3

Page 37: PRESTORMING 2021 SET 2 - TEST 15 HISTORY III - EXPLANATION

EXPLANATION:

Vihara was an early type of Buddhist monastery consisting of an open court surrounded by open

cells accessible through an entrance porch. The viharas in India were originally constructed to

shelter the monks during the rainy season when it became difficult for them to lead the

wanderer‘s life. They took on a sacred character when small stupas and images of the Buddha

were installed in the central court.

Matha, in Hinduism, is any monastic establishment of world renouncers or sannyasis. The first

mathas were founded by the great teacher Adi Shankara in the 8th century. Adi Shankara was

said to have established four mathas at strategic points in India as bulwarks for Hindu

missionary activity

Basadis are Jain monastic establishments or temples in Karnataka. Examples are Neminatha

basadi at Malkhed, Chandragupta Basadi at Shravana Belagola, etc.

Mandapa is an important architectural feature of the Hindu temples which may be a portico or

colonnaded hall that incorporates space for a large number of worshippers. Dances and such

other entertainments are practiced here. Some temples have multiple mandapas in different sizes

named Ardhamandapa, Mandapa, and Mahamandapa.

Hence only Viharas, Mathas, and Basadis denote monastic establishments of different religious

affiliations. So Option (d) is correct.

ADDITIONAL INFORMATION:

Chaityas:

A chaitya is a Buddhist shrine or prayer hall with a stupa at one end for congregational worship by

the Buddhist monks

The chaitya is entered from one end, and at the other end, a small stupa is situated

Chaityas were commonly part of a monastic complex, the viharas

They also served as assemblies for discussions

The Chaitya was divided into three parts and had an apsidal ending, i.e., a semicircular rear end

The central part of the hall also called the nave, was separated from the two aisles by two rows of

pillars

The chaityas also had polished interior walls, semicircular roofs and horse-shoe shaped windows

called the Chaitya windows

Q.46) Match List I with List II and select the correct answer by using the Codes given below the

lists:

List I List II

A. Nayana 1. Jaimini

B. Vaiseshika 2. Kapila

Page 38: PRESTORMING 2021 SET 2 - TEST 15 HISTORY III - EXPLANATION

C. Sankhya 3. Kanada

D. Mimamsa 4. Gautama

Codes:

A B C D

(a) 3 4 2 1

(b) 2 3 1 4

(c) 4 3 2 1

(d) 4 1 3 2

EXPLANATION:

Nyaya philosophy was founded by Gautama Muni. It states that nothing is acceptable unless it

is in accordance with reason and experience. It promotes logical thinking.

Kanada was the founder of the philosophy of Vaiseshika. According to it all objects in the

physical universe are reducible to a finite number of atoms and Brahman is regarded as the

fundamental force that causes consciousness in these atoms. Thinkers of this philosophy believe

that all objects of the universe are composed of five elements–earth, water, air, fire, and ether.

Samkhya is the oldest of the orthodox philosophical system which was introduced by Kapila. It

postulates that everything, in reality, stems from purusha (self, soul, or mind) and prakriti

(matter, creative agency, energy). Purush cannot be modified or changed while prakriti brings a

change in all objects.

Jaimini founded the Hindu orthodox philosophy of Purva Mimamsa. This philosophy states that

Vedas are eternal and possess all knowledge, and religion means the fulfillment of duties

prescribed by the Vedas. It also says that the essence of the Vedas is dharma.

Hence the correct code is A-4, B-3, C-2, D-1. So Option (c) is correct.

ADDITIONAL INFORMATION:

Other Orthodox Schools of Indian Philosophy:

Yoga:

The founder of this philosophy is Patanjali

It means the union of two principal entities

Yogic techniques control body, mind & sense organs, thus considered as a means of

achieving freedom or Mukti

Vedanta:

The founder of this philosophy is Badrayana

Page 39: PRESTORMING 2021 SET 2 - TEST 15 HISTORY III - EXPLANATION

It is also called Uttara Mimamsa

It emphasises on philosophical teachings of the Upanishads (mystic or spiritual

contemplations within the Vedas), rather than the instructions for ritual and sacrifice/

Brahmanas

Later on, this school separated into six sub-schools

Q.47) Which one of the following inscriptions mentions the names of both Chandragupta Maurya

and Asoka?

(a) Shahbajgarhi rock edict

(b) Maski edict

(c) Junagarh inscription of Rudradaman

(d) Junagarh inscription of Skandagupta

EXPLANATION:

The Junagadh rock inscription of Rudradaman is located near Girnar hill near Junagadh in Gujarat.

Hence it is also known as the Girnar Rock inscription of Rudradaman. It consists of Sanskrit prose

inscribed on a rock by the Western Satraps ruler Rudradaman I, written in the mid-second century AD.

It mentions that one of Chandragupta Maurya‘s governors, Pushyagupta, was responsible for building a

dam on Sudarshana Lake near Girnar in Kathiawar. The Sanskrit edict also notes the repairs done by

Mahakshtrap Rudradaman on Sudarshana Lake. It mentions a Yavana king named Tushaspha building

conduits during the reign of Ashoka. The Junagadh rock contains inscriptions of Ashoka, which is one of

fourteen of the Major Rock Edicts of Ashoka.

Hence the Junagarh inscription of Rudradaman mentions the names of both Chandragupta Maurya and

Ashoka. So Option (c) is correct.

ADDITIONAL INFORMATION:

Shahbazgarhi rock edict:

The Shahbazgarhi rock edicts are cut into the surface of two large boulders on the side of a small

rocky outcrop in the Vale of Peshawar

It is located in Peshawar in Pakistan

They are written from right to left in the Kharosthi script

The fourteen major edicts recorded at the site present aspects of Asoka‘s dharma or righteous law

Maski Edict:

It is located in the Raichur district of the state of Karnataka

It consists of a minor rock edict of Emperor Ashoka

Page 40: PRESTORMING 2021 SET 2 - TEST 15 HISTORY III - EXPLANATION

It was the first edict of Emperor Ashoka that contained the name Asoka in it instead of the earlier

edicts that referred to him as Devanampiye piyadasi

The edict is etched on a rock-face of Durgada-gudda, one of the gneissic outcrops that are present

in the site

Junagarh inscription of Skandagupta:

It dates back to the period 450 AD

It states that after his father's death, Skandagupta became the ruler of the earth by his prowess

The inscription states that Skandagupta appointed governors of all provinces, including Parnadatta

as the governor of Saurashtra

It also states that the Sudarshana Lake was repaired during the rule of Skandagupta by his

governor Parnadatta

Q.48) The existence of store houses as famine relief measure in the Mauryan period is known from

(a) Rock edict II from Girnar

(b) Pillar edict II

(c) Minor edict from Sasaram

(d) Sohgaura plate inscription

EXPLANATION:

The Sohgaura plate inscription is inscribed on a copper plate. It is written in Prakrit in Brahmi script. It

was found on the banks of the Rapti River in Gorakhpur District, Uttar Pradesh. It is one of the earliest

known copper-plate inscriptions and is a Mauryan record that mentions famine relief efforts. The text of

the inscription refers to some famine relief measures and notifies the establishment of two public

storehouses at a junction of three great highways of vehicular traffic to meet the needs of persons

(apparently merchants and metal-workers) using these roads.

So Option (d) is correct.

ADDITIONAL INFORMATION:

Rock edict II from Girnar:

It is the Junagadh rock inscription of Rudradaman

It consists of Sanskrit prose inscribed on a rock by the Western Satraps ruler Rudradaman I,

written in the mid-second century AD

It mentions that one of Chandragupta Maurya‘s governors, Pushyagupta, was responsible for

building a dam on Sudarshana Lake near Girnar in Kathiawar

Page 41: PRESTORMING 2021 SET 2 - TEST 15 HISTORY III - EXPLANATION

The Sanskrit edict also notes the repairs done by Mahakshtrap Rudradaman on the Sudarshana

Lake

It mentions a Yavana king named Tushaspha building conduits during the reign of Ashoka

Pillar Edict II:

It is one among the seven pillar edicts of Mauryan king Ashoka

It is written in Prakrit in Brahmi script

It defines Dhamma as the minimum of sins, many virtues, compassion, liberty, truthfulness, and

purity

Minor Edict from Sasaram:

It is located in the state of Bihar

It is written in Prakrit in Brahmi script

It is the Minor Rock Edict I

It indicates that Ashoka turned towards Buddhism gradually, after 2.5 years in power

In this Edict, Ashoka describes himself as a Buddhist layman/Upasaka and also explains he has

been getting closer to the Sangha and has become more ardent in the faith

Q.49) The Gupta gold and silver issue were initially based on the coins of the

(a) Romans and the Saka kshatrapas

(b) Kushanas and Yaudheyas

(c) Kushanas and Saka Kshatrapas

(d) Romans and Kushanas

EXPLANATION:

Gupta coinage started out imitating that of the mighty Kushans and Saka Kshatrapas but very soon had

their own identity which in turn became a forerunner for the dynasties and kingdoms to come thereafter.

Gupta coinage reached the height of metallurgy and iconography. After Indo-Greek Saka Kshatrapa and

Kushan coins, Gupta coinage made a huge come back with a true Indian taste in it. Being indigenous,

they portrayed the king, queen, and deities with Indian attire and with lots of grace.

The Gupta monarchs were famous for their gold coins. They also issued silver coins. However, coins

made of copper, bronze, or any other alloy metals are scarce. The abundance of gold coins from the

Gupta era has led some scholars to regard this phenomenon as the ‗rain of gold‘.So Option (c) is

correct.

Page 42: PRESTORMING 2021 SET 2 - TEST 15 HISTORY III - EXPLANATION

ADDITIONAL INFORMATION:

Gupta Coins:

The Gupta gold coin is known as Dinaras

The gold coins of the Gupta rulers are extraordinary examples of artistic excellence

The coins depicted the ruling monarch on the obverse and carried legends with the figure of a

goddess on the reverse

The images on the coins celebrated the martial qualities and the valor of the ruler

The king-queen types of coins were issued by Chandragupta I, Kumaragupta I, and Skandagupta

The Asvamedha or horse-sacrifice coins were issued by both Samudragupta and Kumaragupta

Goddess depicted in these coins were Laxmi, the Hindu goddess of wealth; Durga, the Hindu

goddess of valor; Ganga, the goddess of the river Ganges; etc.

Some of the Gupta coins, mainly the silver ones, carried the images of Garuda, a mythical bird of

Hindu tradition

Samudragupta issued 8 different types of coins

Q.50) What was considered as Varna Samkara?

(a) Illegitimate Birth

(b) Revolt against the Varna system

(c) Transgression of Varna boundaries for marriage

(d) Championing the Varnasrama dharma

EXPLANATION:

Varna Sankara meant mixing of Varnas or mixed castes. This happened because of intercaste marriages

between various Varnas. This lead to the origination of various numerous Jatis or groups. People

engaged in it were considered ritually impure and the children born out of such marriages were also

considered unwanted, impure, and discriminated against in the society.

Hence Transgression of Varna boundaries for marriages was considered as Varna Samkara. So

Option (c) is correct.

ADDITIONAL INFORMATION:

Anuloma Marriage Pratiloma Marriage

It refers to the Hypergamy form of

marriage

It refers to the Hypogamy form of Marriage

Anuloma marriage is a social practice

according to which a boy from upper

varna/caste/class marries a girl from

Pratiloma is a type of marital practice in

which a man of lower class/caste/varna

marries a girl of higher class/caste/varna

Page 43: PRESTORMING 2021 SET 2 - TEST 15 HISTORY III - EXPLANATION

lower varna /caste/class

Anuloma marriage was considered

preferable to pratiloma marriage

Pratiloma marriage was very much

discouraged and even condemned

Q.51) In Medieval India, the term “bandagan” referred to

(a) Factory producing things used by royals and nobles

(b) Land revenue collected by the Sultan on produce cultivated in the land of the Sultanate

(c) Accountants appointed by the state to check the amount of revenue collected

(d) Special slaves purchased for military service by Iltutmish

EXPLANATION:

Bandagan is a Persian term used for special slaves who are purchased for military service by Delhi

Sultans, especially Iltutmish. Bandagans were carefully trained to man some of the most important

political offices in the kingdom. Since they were totally dependent upon their master, the Sultan could

trust and rely upon them.

Hence the term ―bandagan‖ referred to special slaves purchased for military service by Itutmish. So

Option (d) is correct.

ADDITIONAL INFORMATION:

Karkhana:

It was a manufacturing centre and workshops for craftsmen under state supervision during the

Sultanate and Mughal periods

It was a factory producing things used by royals and nobles

Kharaj:

It was the land revenue collected by the Sultan on produce cultivated in the land of the Sultanate

It was fixed at half of the produce and assessed on the basis of measurement

Accountants:

They were appointed by the state of Sultans to check the amount of revenue collected by the

muqtis

There was proper control regarding the muqti collecting only the taxes prescribed by the state and

that he kept the required number of soldiers.

Page 44: PRESTORMING 2021 SET 2 - TEST 15 HISTORY III - EXPLANATION

Q.52) The iqta under the Delhi Sultans was a territorial assignment and its holder was designated

muqti who

(a) Had no claims on the peasant other than the due land tax

(b) Was the owner of the iqtaland

(c) Was entitled to claim forced labour from the cultivators

(d) Has claim on the person and property of the peasants

EXPLANATION:

The Iqtadari was a unique type of land distribution and the administrative system evolved during the

sultanate of Iltutmish. Under this system, the entire empire was very evenly divided into several large

and small tracts of land called the Iqtas. Most of the Delhi Sultans, like the Khalji and

Tughlaq monarchs, appointed military commanders as governors of these various size territories. These

lands were called Iqta, and their holder was called Iqtadar or Muqti. The duty of the Muqtis was to lead

military campaigns and maintain law and order in their Iqtas. The Muqtis collected the revenues of their

assignments as salary, and they had no claims on the property of the peasants. So Option (a) is

correct.

ADDITIONAL INFORMATION:

Mansabdari System:

It was introduced by Mughal emperor Akbar as new administrative machinery and revenue system

Akbar had borrowed this mansabdari system from Mongolia

Every official in the Mughal empire, high or low, had a rank or a mansab

The Mansabdars were appointed to all civil and military posts

They were liable to be transferred from one branch of the administration (civil) to another (military)

Higher the Mansab, higher the salary, status, and position of the officer

Initially, a single number represented the rank, salary, and the size of the contingent of the

mansabdar

Later, the rank of mansabdar came to be denoted by two numbers – Zat and Sawar

Zat – This indicated the Rank in the administration as well as the salary of the Mansabdar

Sawar – This represented the Cavalry Rank. It denotes the number of horses and cavalrymen

maintained by Sawar

Q.53) Which one of the following sultans extended agricultural loans called sondhars to the poor

peasants for promoting cultivation?

(a) Balban

(b) AllauddinKhalji

(c) Mohd. Bin Tughlaq

(d) FiruzTughlaqAbulFazl

Page 45: PRESTORMING 2021 SET 2 - TEST 15 HISTORY III - EXPLANATION

EXPLANATION:

Muhammad Bin Tughlaq introduced advanced agricultural loans, called Sondhar, to dig wells and to buy

seeds and implements. It was formulated as the first systematic policy of state agricultural loans. It was

offered on a grand scale. However, most of these turned out to be insolvent and became a failure adding

a burden on the finances of the state. The Sondhar loans were publicly written off and the more

oppressive taxes were lifted by Firoz Shah Tughlaq. So Option (c) is correct.

ADDITIONAL INFORMATION:

Muhammad Bin Tughlaq:

He was the most learned Muslim ruler who was well versed in various branches of learning

including astronomy, mathematics and medicine

He was known as the ‗Wisest Fool‘

He transferred the capital from Delhi to Daulatabad

He taxed the farmers of the Ganga-Yamuna doab

He introduced agricultural loans called Taqavi loans

A separate department for agriculture called as Diwan-i-Kohi was created

He made a famine law/code to prevent famines and take relief measures

He introduced token currency made of copper which at par in value to that of the silver coins,

which increased the easy forgery of currency by many

Q.54) Which one of the following statements regarding GhiyasuddinTughlaq‟s concession to the

rural intermediaries is correct?

(a) He allowed them to impose taxes on peasants

(b) He gave them revenue assignments

(c) He enrolled them into the Delhi Sultanate Anny

(d) He exempted their lands from tax

EXPLANATION:

Khots, muqaddams and chaudhuris together formed the rural aristocracy/intermediaries. They belonged

to the highest stratum of the peasantry. Ghiyasuddin Tughlaq introduced moderation. The exemption

from grazing as well as tax on their own cultivation was granted. The system of taxation introduced by

Alauddin seems to have lasted for long though Ghiyasuddin Tughlaq (1320-25) modified it to some

extent and exempted the khots and muqaddams from paying tax on their cultivation and cattle. But the

intermediaries were not allowed to impose any cess upon the peasantry. So Option (d) is correct.

Page 46: PRESTORMING 2021 SET 2 - TEST 15 HISTORY III - EXPLANATION

ADDITIONAL INFORMATION:

Ghiyasuddin Tughlaq:

He was the first ruler and the founder of the Tughlaq dynasty

He was also called as Ghazi Tuglaq

He founded the city Tughlaqabad

In 1323, he crushed the leader of Warrangal and annexed his region

He re-established the food laws of Ala-ud-Din

He crushed the revolts in the unfriendly provinces with sturdy influence and resorted to harmony,

law and order

An improved postal system was structured by him

He encouraged agriculture

Q.55) The term Khalisa in Mughal administration signified the

(a) Land owned by the Emperor himself

(b) Entire imperial Establishment

(c) Land from where revenue was collected for the imperial

(d) Religious land grants

EXPLANATION:

The term Khalisa was used in the Delhi Sultanate and the Mughal administration. It was used to

refer to the Crown Lands. These were lands owned directly by the emperor. Tax collection of

these lands was directly overseen by Wazir of the empire and the revenue went straight into the

central treasury. So Option (a) is correct.

ADDITIONAL INFORMATION:

The Delhi Sultans introduced reforms in the land revenue administration. The lands were classified

into three categories:

Iqta land - Lands assigned to officials as iqtas instead of payment for their services

Khalisa land – Lands under the direct control of the Sultan and the revenues collected were

spent for the maintenance of royal court and the royal household

Inam land – Lands assigned or granted to religious leaders or religious institutions

Q.56) The greatest painter of birds at Jhangir Court was

(a) KhwajaAbdusSamad

(b) Syed Ali Tabrizi

(c) Basaran

(d) Mansur

Page 47: PRESTORMING 2021 SET 2 - TEST 15 HISTORY III - EXPLANATION

EXPLANATION:

The greatest painter of birds at Jahangir's court was Ustad Mansur. Mansur was the leading nature

painter at the court of the Mughal emperor Jahangir. Mansur was extraordinarily talented in scientific

documentation. He was extremely talented and valued for his immaculate depiction of the natural world.

His detailed careful depictions of plants and animals avoided all personal expression and are extremely

valuable for their scientific accuracy as well as their artistic perfection. Jahangir bestowed the title of

Nadir-ul-Asr (Miracle of the Age/ Wonder of the Age) on Ustad Mansur. Mansur was assigned to

document all plants and animals during trips of the emperor whom he accompanied on some of his

travels. Falcon Perched on a Bird Rest (1615), is one of the many paintings that were painted by Ustad

Mansur.

So Option (d) is correct.

ADDITIONAL INFORMATION:

Jahangir:

Jahangir was the eldest son of Akbar

His original name was Nur-ud-din Muhammad Salim

He assumed the title of Jahangir (conqueror of the world) when he assumed the Mughal throne

He served as the Mughal emperor of India from 1605-1627

He was especially fond of art and architecture and commissioned, catalogued and preserved many

artworks

The Mughals paintings reached its zenith under him

He also kept a zoo where he documented each species of animal

He was visited by Sir Thomas Roe, the first English ambassador to the Mughal court

Jahangir, like his father, was not a strict Sunni Muslim; he allowed, for example, the Jesuits to

dispute publicly with Muslim Ulemas (theologians) and to make converts

Q.57) Which one of the following is correct about the painters attached to the imperial

establishment during the Mughal period?

(a) They did not depict the common man and his life

(b) They never showed any technological devices in their paintings

(c) They never made paintings as joint work

(d) They made self portraits

EXPLANATION:

Paintings of the time depicted the social contract between royal families and commoners through the

scenes of courts where the distance between ruled and rulers differed in each painting of different rulers.

Page 48: PRESTORMING 2021 SET 2 - TEST 15 HISTORY III - EXPLANATION

Hence one can see how close a ruler was with his subjects or how adored he was by the common people.

There was a depiction of hunting scenes and celebration of festivals like Holi, Diwali. Hence it depicted

the common man and his life.

Mughal paintings show the construction scenes of the big monuments and also tell us about the used

technologies. For instance, some paintings show us how stones were cut and polished to be used in

Fatehpur Sikri. Hence use of technological devices is visible in their paintings.

The artworks produced during the early Mughal phase were collaborative efforts of the team of artists.

And based on one‘s specialisation, each artist would undertake an aspect of the painting that he was

comfortable with or delegated. Hence they made paintings as joint works.

Imperial portraits with glorious titles were painted to present the personality of the emperor himself. The

best example of this period is the portrait of Jahangir himself, who is shown holding a picture of the

Virgin Mary in his right hand. Hence they made self-portraits.

ADDITIONAL INFORMATION:

Mughal Paintings:

Mughal painting is the style of miniature painting that developed in the northern Indian

subcontinent in the sixteenth century and continued till the mid-nineteenth century

Mughal paintings display a mixture of Indian, Persian and Islamic styles

The royal court scenes, portraits, depiction of accurate flora and fauna were some of the favourite

themes of the Mughal artists

Most of the paintings were parts of manuscripts and royal albums

When Humayun returned to India from the outcast, he additionally brought along two eminent

Persian artists, Mir-Sayyid Ali and Abd-us-samad

Mughal paintings advanced and flourished under the reign of Mughal Emperors, Akbar, Jahangir

and Shah Jahan

They had fine and delicate drawings along with calligraphic text descriptions, generally on border

Once the painting was complete, agate, a gemstone, was used to burnish the work to set the

colours and give desired radiance to the painting

Gold and silver powders were mixed with colours or sprinkled to add extravagance to a painting

Q.58) Akbar‟s buildgs at FatchpurSikri are famous for

(a) Synthesis of trabeate and arcuate styles

(b) Construction of the first bulbous dome in India

(c) The use of marble on an extensive scale

(d) Irequent employment of pietra dura

Page 49: PRESTORMING 2021 SET 2 - TEST 15 HISTORY III - EXPLANATION

EXPLANATION:

Fatehpur Sikri was a town planned as an administrative unit consisting of public buildings as well as a

private residence in close proximity. Buildings in Fatehpur Sikri are made of red sandstone with a

combination of trabeated and arcuate styles of construction and restricted ornamentation. Fatehpur

Sikri also presents an amalgamation of the Timurid and Central Asian architectural styles with the

indigenous architecture of India.

In ―trabeate‖ or ―corbelled‖ style of architecture, roofs, doors and windows were made by placing a

horizontal beam across two vertical columns. In ―arcuate‖ form of architecture the weight of the

superstructure above the doors and windows was carried by arches.

So Option (a) is correct.

ADDITIONAL INFORMATION:

Fatehpur Sikri:

It is located near Agra in Uttar Pradesh

The construction of the city was begun in 1569 and completed in 1574

Akbar founded the city in 1569 after winning Chittor and Ranthambore

The architecture of the city was designed by Tuhir Das and Dhruv Chawla

Persian and Indian architecture was used in the construction of the city

The city is a modest and compact township, consisting of halls, palaces, offices, gardens, pleasure

resorts, baths, mosques, tombs

The pavilions of the imperial palace are geometrically arranged and this design was adopted from

Arabian architecture

At Fatehpur Sikri many of the buildings show the influence of the architectural styles of Gujarat

and Malwa

Akbar abandoned the city in 1585 due to the shortage of water and frequent wars between

Mughals and Rajputs

Q.59) Which one of the following places has yielded archaeological evidence regarding pit

dwellings?

(a) Rakhigarhi

(b) Ghufkral

(c) Mehrgarh

(d) Balakot

EXPLANATION:

The evidence of pit-dwellings has been discovered from Burzahom and Gufkaral. Both these

archaeological sites belong to the Neolithic period. They are located in Jammu and Kashmir. Gufkral is

Page 50: PRESTORMING 2021 SET 2 - TEST 15 HISTORY III - EXPLANATION

the northernmost excavated Neolithic site of India.

So Option (b) is correct.

ADDITIONAL INFORMATION:

Pit-dwellings:

The pit-dwelling of Neolithic men were cut into the natural soil usually dug out with long stone

celts, the cuts-marks of which can still be traced

The pits were circular or oval in plan, narrow at the top and wide at the base having (wooden) post

holes on the ground level suggesting a birch cover as a protection against the harsh weather

Some pits were shallower, with the depth of about 91 cm (as opposed to 3.95 meters depth) and

were possibly either storage pits or those used as dwellings during the warmer period

Stone hearths have also been found at ground levels; near the mouth of pits, showing that

habitation activities were also at the ground level

Ascribed to the same era are subterranean dwellings of the quadrangular section, covered by a

layer of birch, with a centrally placed stone or clay hearth and storage pit

Q.60) The crops cultivated by the Harppans included

(a) Wheat, barley and mustard

(b) Baley, groundnut and rice

(c) Whea, rice and sugarcane

(d) Wheat, cotton and sugarcane

EXPLANATION:

The crops cultivated by the Harappans were:

Wheat

Barley

Rice

Cotton

Peas

Sesame

Linseed

Mustard

Chickpea

Millets

The crops like groundnuts, sugarcane, tomatoes, potatoes, etc., were not known to them. So Option (a)

is correct.

Page 51: PRESTORMING 2021 SET 2 - TEST 15 HISTORY III - EXPLANATION

ADDITIONAL INFORMATION:

Crops that diffused from West Asia:

Wheat

Barley

Cotton

Crops that diffused from Africa:

Ragi

Bajra

Q.61) Match List I with List II

List I List II

A. Double burial 1. Chanhudaro

B. Fire altars 2.Kalibangan

C. Workmen‘s quarters 3. Lothal

D. Bead making 4. Banawali

5. Harappa

Codes

A B C D

(a) 3 2 5 1

(b) 3 4 1 2

(c) 4 2 3 5

(d) 2 3 5 1

EXPLANATION:

Double burial/Twin burial was found in Lothal in Gujarat. This practice involved the burial of a

male & female person in a single grave.

The worship of fire in Harappan Civilisation is proved by the discovery of fire altars at Lothal and

Kalibangan. Fire altars were found here in forts & residences. There were platforms in residences

which used to support the fire altars. The altars were arranged in a row and there were stairs to

access those fire altars.

Workmen quarters near the granary at Harappa proved the presence of a class of workmen. It

also showed they considered the welfare of workers.

Page 52: PRESTORMING 2021 SET 2 - TEST 15 HISTORY III - EXPLANATION

Bead making was found in the Harappan site of Chanhudaro which is located in Pakistan.

Materials for making beads included beautiful red coloured stone-like carnelian, jasper, crystal

quartz and steatite.

So Option (a) is correct.

ADDITIONAL INFORMATION:

Findings Harappan Site

Dockyard Lothal

Dancing girl statue Mohenjadaro

Furrowed land Kalibangan

Cemetery Harappa

Pot Burial Surkotda

Q.62) Which of the following Harappan sites are located in the Kutch region?

(a) Desalpur and Surkotada

(b) Rangpur and Rojidi

(c) Allabadino and Balakot

(d) Lothal and Surkotada

EXPLANATION:

Desalpur is a disproportionately and largely fortified Harappan site located in Nakhtrana taluka in the

Kutch district of Gujarat. Surkotda is located in Rapar taluka of Kutch district in Gujarat. At Surkotada,

a compact citadel and residential annexe complex has been found.

Rangpur is an ancient archaeological site near Vanala on the Saurashtra peninsula in Gujarat .It is a

Harappan site lying on the tip between the Gulf of Khambat and Gulf of Kutch. Rojidi is located on the

northern bank of the Bhadar river in Gondal taluka of Rajkot district in central Saurashtra peninsula of

Gujarat. Mostly Harappan pottery was found here.

Allahdino is a Harappan site located in the Karachi district of Pakistan. Balakot is located in Mansehra

district of Pakistan and has remains of terracotta figurines.

Lothal is located in the Bhal region of Gujarat. Here double burial, dockyard, etc., have been found.

Hence Desalpur and Surkotada are Harappan sites located in the Kutch region. So Option (a) is

correct.

Page 53: PRESTORMING 2021 SET 2 - TEST 15 HISTORY III - EXPLANATION

ADDITIONAL INFORMATION:

Important Harappan sites:

Mohenjadaro- Pakistan

Harappa - Pakistan

Kalibangan - Rajasthan

Dholavira - Gujarat

Lothal - Gujarat

Surkotada - Gujarat

Rakhigarhi - Haryana

Banawali - Haryana

Chanhudaro - Pakistan

Sutkagendor - Pakistan

Amri - Pakistan

Q.63) „Sutudri‟ mentioned in the Rig Veda is

(a) A term used to denote bride

(b) The name of a river

(c) A plant used in the preparation of Sura

(d) A sacrificial animal

EXPLANATION:

Sutudri mentioned in the Rig Veda is the name of a river. It denotes the Sutlej River which is the

easternmost tributary of the Indus river. It starts in the Tibetan region of China and then flows through

India and drains into the sea after passing through Pakistan.

So Option (b) is correct.

ADDITIONAL INFORMATION:

Rigvedic Rivers:

Old Name Present Name

Sindhu Indus

Vitasta Jhelum

Askini Chenab

Purushni Ravi

Vipas Beas

Page 54: PRESTORMING 2021 SET 2 - TEST 15 HISTORY III - EXPLANATION

Drishdvati Ghagghar

Suwastu Swat

Q.64) Which one of the following is correctly matched in the context of the Vedic culture?

(a) Asura: A sacred plant

(b) Pushan: Tribal assembly

(c) Rita: Inebriating drink

(d) Ribhu: Gnomes

EXPLANATION:

The term Asura appears first in the Vedas and refers to a human or divine leader. Its plural form

gradually predominated and came to designate a class of beings opposed to the Vedic gods.

Pushan was the god of marriages and was also referred to as the god of meeting. Pushan was

said to be responsible for marriages, roads, journeys and the feeding of cattle and is a Vedic solar

deity and one of the twelve Adityas. Pushan was also regarded as the god of Shudras.

Rita is a Rig Vedic concept being repeatedly mentioned in Rig Vedas. Rita refers to the physical

order of the universe, the order of the sacrifice, and the moral law of the world. It led to the

doctrines of dharma and karma.

In early times of Vedic culture Ribhus are referred to a sun deity. They are referred to three male

artisans whose abilities and austerities make them into divinities in later Vedic texts. They are

famous for their creative abilities, innovation and they design chariots, the magic cow of plenty,

channels for rivers, and tools for Indra and other gods, which makes many envious. They can be

referred to as gnomes. So Option (d) is correct.

Q.65) Which one of the following Vedangas is not correctly matched?

(a) Kalpa: Ritual

(b) Jyotisha: Astronomy

(c) Siksha: Phonetics

(d) Nirukta: Grammar

EXPLANATION:

Kalpa deals with rituals. It contains sacrificial practice and systematic sutras. Kalpa has 4 kinds

of sutras which are Srautasutras, Smartasutras, Grhyasutras and Dharmasutras.

Jyotisha is associated with astronomy or astrology. It describes rules for tracking the motions of

the sun and the moon and the foundation of Vedic Jyotish. Maharshi Lagadha is regarded author

of this Vedanga Jyotisha.

Shiksha deals with phonetics, pronunciation and accent. It explains how each syllable should be

pronounced in which context and the psychic effects of those. Pratisakya suthra is the earliest

Page 55: PRESTORMING 2021 SET 2 - TEST 15 HISTORY III - EXPLANATION

book available on Shiksha.

Nirukta deals with etymology, the origin of the meaning of the word, particularly of obscure

words, especially those occurring in the Veda. Sanskrit grammarian Yaska is regarded as the

author of Nirukta. It does not deal with grammar.

So Option (d) is correct.

ADDITIONAL INFORMATION:

Other Vedangas:

Vyakaran:

It deals with grammar

Panini‘s Ashtadhyayi was the earliest known grammar book

Chhanda:

It is the science of metres/metrics

It measures and divides Vedic Mantras by number of padas in a verse

Acharya Pingala is the author of this vedanga

Q.66) Which of the following characterized the latest Vedic period?

1. Extensive bruing of forests

2. Manufacture of iron artifacts

3. Knowledge of seasons

(a) 1, 2 and 3

(b) 1, 2, and 4

(c) 1, 3 and 4

(d) 2, 3 and 4

EXPLANATION:

During the Later Vedic Age, the Aryans thoroughly subdued the fertile plains watered by rivers

like Yamuna, Ganges and Sadanira. It marked the extensive burning of forests. The purpose of

burning the forests was to create land for agriculture.

This period had extensive use of iron and the manufacture of iron artefacts and tools. These iron

tools helped people in clearing the forests and bringing more land under cultivation for

agriculture.

During the later-Vedic period, a distinct advance was made in the science of astronomy which

was associated with local climate, agriculture, etc. This is evident from the fact that the Vedas

introduced the Nakshatras and mention the six seasons. Barley was sown in winter, ripened in

summer and rice was sown in the rainy season and was harvested in autumn.

So Option (a) is correct.

Page 56: PRESTORMING 2021 SET 2 - TEST 15 HISTORY III - EXPLANATION

ADDITIONAL INFORMATION:

Later Vedic Period:

It is characterised by the period of 1000-600 B.C

In this period larger kingdoms were formed as many janas or tribes amalgamated to form

janapadas or rashtras

The king performed various rituals and sacrifices to strengthen his position like Rajasuya,

Asvamedhs and Vajpeya

In addition to existing purohita, senani and gramani, new officials like treasury officer, tax collector

and royal messenger were included in the administration

The importance of Sabhas and Samitis had diminished

The 4 divisions or the varna system was thoroughly established

The position of women declined

Besides nishka, gold and silver coins like satamana and krishnala were used as the medium of

exchange

Prajapati, Vishnu and Rudra became the prominent gods

Q.67) Rig Vedic period invoked India for context of the Vedic culture?

(a) Acquisition of knowledge

(b) A life after death

(c) Release from the cycle of births and deaths

(d) Material comforts and victory

ADDITIONAL INFORMATION:

Important Rig Vedic Gods:

Agni:

Personification of Fire

Agni was second in importance only next to Indra

He acted as the intermediary between the Gods and the worshipper

Varuna:

Personification of Rain

God of truth and moral order

Prithvi:

Personification of Earth

Goddess of grain and procreation

Vayu :

Personification of Wind

Soma:

God of plants

Surya: Destroyer of darkness

Maruta:

God of storm

Page 57: PRESTORMING 2021 SET 2 - TEST 15 HISTORY III - EXPLANATION

Q.68) Which of the following are characteristics ascribed to Buddhism? Chose the correct answer

given below

1. Rejection of the authority of the Vedas

2. Emphasis on the role of the individual

3. Belief in the categories of jiva and ajiva

4. Theory of Prakriti and Purusha

(a) 1 and 2

(b) 1, 3 and 4

(c) 2 and 3

(d) 3 and 4

EXPLANATION:

Buddha repudiated the thesis that the Vedas are infallible and their authority could never be

questioned. In his opinion, nothing was infallible and nothing could be final. Everything must be open to

re-examination and reconsideration. Buddhism is considered a Nastika school since it rejects the

authority of the Vedas.

Buddhism has a strong individualistic component that everyone has responsibility for their own

happiness in life. Buddhism‘s individual outlook and disregard for the caste system in attaining

enlightenment were appealing to people in lower castes. Buddhism suggested that individual people

might be able to attain enlightenment in this life and held that caste was not a punishment for deeds

committed in a past life

The Buddhist concept of anatta or anatman is one of the fundamental differences between mainstream

Buddhism and mainstream Hinduism, with the latter asserting that atman (self, soul) exists. Anatta in

Buddhism, means the doctrine that there is in humans no permanent, underlying substance that can be

called the soul. Instead, the individual is compounded of five factors that are constantly changing. Hence

Buddhism does not believe in the categories of jiva and ajiva.

Sankhya philosophy tells us that the universe was born from the union of Prakriti and Purusha. Prakriti

here refers to the basic cosmic material that is the root of all beings, and Purusha to the spirit or

conscious energy that governs life and reality. The cosmic entity, Purusha, exists beyond the realms of

time and space and combines with Prakriti to form this world of creation. Both these eternal

indestructible realities join to form the matter we perceive. Buddhism holds to the theory of an ever

changing universe. But their change has no background. Every change is absolutely a new one. Hence

Buddhism does not believe in the theory of Prakriti and Purusha.

Hence only 1 and 2 are correct. So Option (a) is correct.

ADDITIONAL INFORMATION:

Eight fold path in Buddhism:

Right view

Right resolve

Right speech

Page 58: PRESTORMING 2021 SET 2 - TEST 15 HISTORY III - EXPLANATION

Right conduct

Right livelihood

Right effort

Right mindfulness

Right samadhi

Q.69) Jivaka mentioned in the early Buddhist literature, was a

(a) Bodhisativa

(b) King

(c) Merchant

(d) Physician

EXPLANATION:

Jivaka was the most celebrated doctor in India during the Buddha's time. He was the royal physician in

the court of Bimbisara. He was called upon to treat kings and princes. But of all the distinguished

people Jivaka attended to, his greatest pleasure was to attend to the Buddha. He was a disciple of

Atreya. When Udyot, the king of Ujjain, suffered from jaundice, then he requested Emperor Bimbisara to

send Jivaka to Ujjain for a few days and Jivaka cured him with his medicines. He was educated at Taxila

University. So Option (d) is correct.

ADDITIONAL INFORMATION:

Sushruta:

He was an ancient Indian physician

He lived in the period of 6th century BCE in the ancient city of Kashi, now known as Varanasi or

Banaras

He is regarded as 'Father of Indian Medicine' and 'Father of Plastic Surgery'

He wrote the book titled Sushruta Samhita which deals mainly with surgical knowledge and is

highly regarded as one of the Great Trilogy of Ayurvedic Medicine

Q.70) Which of the following terms are used to denote Jaina texts?

1. Nirgranthas

2. Purvas

3. Angas

4. Upangas

Page 59: PRESTORMING 2021 SET 2 - TEST 15 HISTORY III - EXPLANATION

(a) 1, 2 and 3

(b) 2, 3 and 4

(c) 1 only

(d) 2 and 4 only

EXPLANATION:

A Nirgrantha is a term that specifically refers to Jains in religious and philosophical works from

the Indian religions. It means they are people who are free from fetters or without any bonds. It

was a title given to ascetics who freed themselves from the knots which mean the household life

of desires, duties and worries. Mahavira is often referred to as Nirgrantha in Buddhist texts.

The teachings of the Tirthankaras before Mahavir are known as Purva. There were total 14

Purvas. The persons having the knowledge of these purvas were given the status of Shrutakevali

or scripturally omniscient persons. Jain Scholar Bhadrabahu, who was also a Sutra Kevali could

recite the 14 Purvas. Bhadrabahu is considered to be the last expert of fourteen Purvas.

The preachings or sermons of Mahaveera were finally compiled in 12 volumes during the second

Jain council. These were called 12 Angas by the Jains. They are in Prakrit language.

The 12 Upangas were compiled in the second Jain council. They serve as explanations to the 12

Angas.

So Option (b) is correct.

ADDITIONAL INFORMATION:

Other Jain texts/Agamas:

Chedasutras:

There are 6 Chedasutras

These are texts related to behaviour of Monks and Nuns

Chulikasutras:

There are 2 Chulikasutras

These are texts which further enhance or decorate the meaning of Angas

Mulasutras:

There are 4 Mulasutras

These are texts which provide a base in the earlier stages of the monkhood

Prakirnakasutras:

There are 10 Prakirnakasutras

These are texts on independent or miscellaneous subjects

Page 60: PRESTORMING 2021 SET 2 - TEST 15 HISTORY III - EXPLANATION

Q.71) Which one of the following philosophies is associated with Jainism?

(a) Syadvada

(b) Yogacara

(c) Madhyamika

(d) Sunyavada

EXPLANATION:

Syadvada is a Jain philosophy. It is the theory of relativity of knowledge. It is the doctrine that all

judgments are conditional, holding good only in certain conditions, circumstances, or senses. It suggests

that the ways of looking at a thing are infinite in number.

Yogacara, Madhyamika and Sunyavada are Buddhist philosophies.

So Option (a) is correct.

ADDITIONAL INFORMATION:

Yogacara:

It is an influential idealistic school of Mahayana Buddhism

Yogacara school developed during the fourth century during Gupta and Pala period

It emphasizes the study of cognition, perception, and consciousness through the interior lens of

meditative and yogic practices

Asanga and Vasubandhu were two philosophers of Yogacara school

Madhyamika:

It is an important school in the Mahayana Buddhist tradition

It takes a middle position between the realism of the Sarvastivada school and the idealism of the

Yogacara school

Nagarjuna and his disciple Aryadeva are the exponents of this philosophy

Sunyavada:

It is a Buddhist philosophy

It was developed by 2nd-century Indian philosopher Nagarjuna

It emerged out of the Madhyamika school and sometimes treated synonymous to it

It states that the voidness constitutes ultimate reality

It is seen not as a negation of existence but rather as the undifferentiation out of which all

apparent entities, distinctions, and dualities arise

Page 61: PRESTORMING 2021 SET 2 - TEST 15 HISTORY III - EXPLANATION

Q.72) Hiuen Tsang found Jainism flourishing in

(a) Orissa

(b) Kashmir

(c) Bengal

(d) Bihar

EXPLANATION:

When the Chinese pilgrim Hiuen Tsang stayed in India he also visited Bengal. He wrote that among the

non-Buddhists in Pundravardhana the majority were Digambara Nirgranthas. Hiuen Tsang states that

in Pundravardhana and Samatata, i.e., in north-western and south-eastern Bengal the naked ascetics

called Nirgranthas are most numerous. So Option (c) is correct.

ADDITIONAL INFORMATION:

Hiuen Tsang:

He stayed in India for 16 years from 629 AD to 645 AD

He is also referred as Xuanzang

The primary aim of the visit of Hiuen Tsang to India was to gain knowledge of Buddhism and

collect its religious texts

He visited India during the period of Harsha

Harsha called a religious assembly at Kannauj to honour Hiuen Tsang and he presided over that

assembly

He remained at the University of Nalanda for about five years

He had also visited South India as far as Kanchi

Hiuen Tsang has written about the Fourth Buddhist council

When he went back to China, he wrote a detailed description of India during the reign of Harsha in

his book ‗Si-yu-ki‘ or ‗Record of the Western Countries‘

Q.73) The planning of the capital in the Arthasastra clearly indicates that the industrial and

commercial classes were

(a) Not given any consideration

(b) Provided specific quarters in the township

(c) Allotted space outside the township

(d) Allotted space inside and outside the township

Page 62: PRESTORMING 2021 SET 2 - TEST 15 HISTORY III - EXPLANATION

EXPLANATION:

According to Arthasastra, the palace should be in the central part. It should face either north or east.

The houses of priests and ministers should be on the south-east, traders, skilled workers, and

kshatriyas on the east, the treasury, goldsmiths and industries on the south, forest produce on the

northeast and doctors , the army commander, artists, on the south. Temples should be located in the

center of the town. Cemeteries should be located on the north and east of the town and that for the

higher caste to be located on the south. The depressed classes should be housed beyond the cemetery.

There should be one well for every group of ten houses.

Hence this shows that the planning of the capital in the Arthasastra clearly indicates that the industrial

and commercial classes were provided specific quarters in the township. So Option (b) is correct.

ADDITIONAL INFORMATION:

Arthasastra:

It was written by Kautilya who was also called as Chanakya or Vishnugupta

It was written during the period of 3rd century BC when Chandragupta Maurya was the ruler

It was written in the Sanskrit language

Kautilya's Arthshastra had a wide influence of Sanskrit literature

Three broad themes covered in it are:

Statecraft

Economic policy

Military strategy

Kautilya's administrative and judicial structure was hierarchical in nature

The king had to be acquainted in the four Vedas and four sciences of government

According to him, there is stability if rulers are responsive, responsible, accountable, removable

and recallable, otherwise there would be instability

Town, which is congested, should be freed of surplus population, which should then be housed in

a new location

Dams should be built over rivers and nalas

Building of roads, connecting the same to the remotest corner and also maintaining its beauty is of

prime importance to Kautilya

Consumer protection is another area which receives elaborate treatment from Kautilya. Black

marketing, adulteration, manipulation in selling goods is highly discouraged

Crisis management is another area in which we find Kautilya was a mastermind especially during

the natural calamities

A lot of emphases has been laid down to provide and protect public amenities in Kautilya‘s

Arthashastra

Page 63: PRESTORMING 2021 SET 2 - TEST 15 HISTORY III - EXPLANATION

Kautilya suggests regular inspections and monitoring of reports

Another name of Kautilya‘s Arthashastra is Dandaniti, meaning the book on the Art of Punishment

Q.74) Which of the following statements regarding the Maurya coins are correct?

1. They are punchmarked coins in silver and copper

2. They bear names of kings who issued them

3. They bear distinct sets of symbols

(a) 1 and 2

(b) 1 and 3

(c) 2 and 3

(d) 1, 2 and 3

EXPLANATION:

The Mauryan Empire used the punch-marked coins but with royal standard to make sure of their

authenticity. The liberty to use two different metals for coins was granted, and hence coins in silver and

copper were very popular. So Statement 1 is correct.

The first coins to bear the names and images of rulers were issued by the Indo-Greeks, who established

control over the north-western part of the subcontinent in the 2nd century BCE. Mauryan coins issued

during the 4th and 3rd century BCE did not bear the names of the kings. So Statement 2 is not correct.

Attempts made to identify the symbols on punch-marked coins with specific ruling dynasties, including

the Mauryas, suggest that these were issued by kings. The motifs found on these coins were mostly

drawn from nature like the sun, various animal motifs, trees, hills, etc., and some were geometrical

symbols. Hence they bear a distinct set of symbols. So Statement 3 is correct.

ADDITIONAL INFORMATION:

Punch-marked coins:

The first documented coinage is deemed to start with Punch-marked coins issued between the 7th -

6th century BC and 1st century AD

These coins are called punch-marked coins because of their manufacturing technique in which

symbols on these were hammered using punches and dies

They were mostly made of silver, and sometimes even copper

They were sometimes square, or round but mostly rectangular in shape

A standard punch-marked coin often weighed around 32 rattis or about 52 grains

The Punch marked coin series can be further divided into four categories: The Taxila-Gandhara

type, the Kosala type, the Avanti type, and the Magadhan type

Page 64: PRESTORMING 2021 SET 2 - TEST 15 HISTORY III - EXPLANATION

Issued initially by merchant Guilds and later by States, the coins represented a trade currency

belonging to a period of intensive trade activity and urban development

They are broadly classified into two periods: the first period (attributed to the Janapadas or small

local states) and the second period (attributed to the Imperial Mauryan period)

Q.75) Which one of the following was a major item of import in India during the post-Gupta

period?

(a) Horses

(b) Leather goods

(c) Medicinal herbs

(d) Silk

EXPLANATION:

In the post-Gupta period, the major item of import was horses. The horses were imported from Central

and Western Asian countries like Arabia, Persia, and Afghanistan. Ivory was imported from Ethiopia.

Chinese silk, which was known as chinasunka, had a good market in India. Leather goods and medicinal

herbs were exported and not imported.

So Option (a) is correct.

ADDITIONAL INFORMATION:

Post-Gupta economy and trade:

The trade with South East Asia during the post-Gupta age registered a significant increase

The Arab, Chinese and Indian sources mention the flow of trade between east and west via India

India exported:

Spices

Sandal wood

Pearls

Camphor

Cotton

Metals

Precious and semi-precious stones

In the post-Gupta age, guilds or association of traders & merchants called as Shrenis were

considered important

The trade with East Africa was mediated by Arabs

The decline of commerce is demonstrated by the paucity of coins in the post-Gupta period

Page 65: PRESTORMING 2021 SET 2 - TEST 15 HISTORY III - EXPLANATION

The post-Gupta period also underwent a major change in the rural sector and that was land grants

on a large scale as donations

Long-distance trade declined and urban centres started disappearing

Q.76) Which one of the following is correctly matched?

(a) Uraiyur - Speices

(b) Korkai - Pearl Fishery

(c) Vanji - Silk

(d) Kanchi - Ivory

EXPLANATION:

Uraiyur served as the capital of early Cholas in the Sangam period. It was an important centre of

the cotton trade. These were in great demand in the western world especially for the cotton

clothes woven at Uraiyur

Korkai is a port town in the Pandya kingdom. It was famous for the Pearl fishery.

Vanji which is considered as the present-day town of Karur was the capital of early Chera kings

of Sangam age. It may have been the centre for old jewellery-making and gem setting (with the

gold imported mainly from Rome), as seen from various excavations. Ptolemy mentioned it as a

very famous inland trading centre in Tamil Nadu.

Kanchi was the capital city of the Pallava kingdom. Silk trade in Kanchipuram began when King

Raja Raja Chola-I invited weavers from Saurashtra, Gujarat to migrate to Kanchi. It was famous

for its temples. It served as a noted centre of learning for Tamil and Sanskrit scholars.

So Option (b) is correct.

EXPLANATION:

Exports in Sangam age:

Salt, pepper, pearls, ivory, silk, spices, diamonds, saffron, precious stones, muslin, sandal wood

Imports in Sangam age:

Topaz, tin, wine, glass, horses, gold

Q.77) Which one of the following inscriptions provides the earliest epigraphical evidence regarding

Sati?

(a) Mathura inscription of Huvishka

(b) Allahabad Pillar inscription of Samudragupata

(c) Jungarh inscription of Skandagupta

(d) Eran Pillar inscription of Bhanugupta

Page 66: PRESTORMING 2021 SET 2 - TEST 15 HISTORY III - EXPLANATION

EXPLANATION:

The Eran Pillar inscription was engraved in 510 A.D. It is located at Eran in Sagar district in Madhya

Pradesh. It is a posthumous inscription of Goparaja. Goparaja, a warrior of Gupta ruler Bhanugupta

came to Eran and lost his life in a great battle against the Hunas. The inscription mentions that his wife

subjected to Sati in her husband's pyre.

So Option (d) is correct.

ADDITIONAL INFORMATION:

Mathura inscription of Huvishka:

Huvishka was a ruler of the Kushana empire

It was inscribed in the year 33 of Huvishka

This inscription is on the broken pedestal of a seated Buddha image from the Chaubara mound

near Mathura

The language is the ordinary mixed dialect of Sanskṛit and Prakṛit

Allahabad Pillar inscription of Samudragupata:

It gives a detailed account of the career and personality of Samudragupta

It was composed by Harisena who was the court poet and minister of Samudragupta

It also mentions about boundaries of the Gupta Empire. It lists 14 kingdoms bordering

Samudragupta‘s kingdom

As per this inscription, Samudragupta defeated 9 kings in North, 12 Kings in South, reduced all

the Atavika states to vassalage

It also mentions Samudragupta as a great musician

Junagarh inscription of Skandagupta:

It dates back to the period of 450 AD

It states that after his father's death, Skandagupta became the ruler of the earth by his own

prowess

The inscription states that Skandagupta appointed governors of all provinces, including Parnadatta

as the governor of Suarashtra

It also states that the Sudarshana Lake was repaired during the rule of Skandagupta by his

governor Parnadatta

Q.78) The post-Gutpa social structure in north India was marked by the

(a) Growing importance of women

(b) Lessening of untouchability

(c) Proliferation of castes

(d) Revival of slavery

Page 67: PRESTORMING 2021 SET 2 - TEST 15 HISTORY III - EXPLANATION

EXPLANATION:

The emergence and proliferation of new castes and the hardening of caste relations was an important

change in the social structure in the post-Gupta period. The period witnessed the ascendancy of

Varnashramadharma. Hiuen Tsang wrote about the existence of four varnas in society. Bana and

Hiuen Tsang convey the existence of many sub-castes.

The position of women seems to have deteriorated during this period. Sati and dowry were common.

Girls were married between the ages of six and eight years. In general, women were distrusted. They

were to be kept in seclusion. Generally, the lives of women were controlled by their male relatives like

son, father, and brother.

The practice of untouchability appears to have intensified and the number of untouchables registered

a rise. Not only hunters and some groups of artisans became untouchables but backward

agriculturists were also condemned to that status. The untouchables did not normally hold land and

settled outside the villages.

So Option (c) is correct.

ADDITIONAL INFORMATION:

Society in the post-Gupta period:

The social structure was broadly characterised by a sizable ruling landed aristocracy,

intermediaries and a large body of impoverished peasantry

The unequal distribution of landed property and power led to the emergence of new social

groups and ranks which cut across varna divisions like Brahmana, Kshatriya, Vaisya and

Sudra

Acculturation of the tribes was also a major change in this period

The land grants facilitated feudal development in India from the fifth century AD onwards

The peasants stayed in the land granted to the feudal lords

The villages transferred to these grantees were termed as Sthana-Jana-Sahita and Janata

Samriddha

The growth of the feudal society weakened the position of the king and made him more

dependent on the feudal chiefs

The domination of the feudal chiefs also resulted into the weakening of the village self-

government

Q.79) Which one of the following pairs is not correctly matched?

(a) Jaunpur : Atala Mosque

Page 68: PRESTORMING 2021 SET 2 - TEST 15 HISTORY III - EXPLANATION

(b) Malwa : Jahaz Mahal

(c) Ajmer : Quuwat-al-Istam

(d) Gulbarga : Jama Masjid

EXPLANATION:

Atala masjid/mosque was built on the site of a Hindu temple of Atala devi at Jaunpur in Uttar

Pradesh during 1408 AD. Hence its name came as Atala masjid. The stone materials of Atala Devi

temple and other nearby temples were utilized in the construction of this mosque.

Jahaz mahal is located in the Mandu Fort in the Malwa region in the state of Madhya Pradesh.

Situated between two artificial lakes, this two-storied architectural marvel is so named as it

appears as a ship floating in water. Built by Sultan Ghiyas-ud-din-Khalji, it served as a harem for

the sultan.

The Quwwat-ul-Islam or Might of Islam Mosque is the earliest surviving mosque in India and

stands in the Qutb Minar enclosure in Delhi. It was begun in 1193 AD by Qutb-ud-Din Aibak,

the first Sultan of Delhi and founder of the Slave Dynasty and completed it four years later, with

further additions made by later Sultans in the 13th and 14th centuries. It was built on the site of

Rai Piathora‘s Hindu temple using the spoil from 27 other Hindu and Jain temples. It is not

located in Ajmer.

Jama Masjid in Gulbarga (Kalaburagi) in the state of Karnataka is known for its unique Spanish

architecture. It was built by Ala-ud-din Hasan Bahmani Shah and opened in 1367 AD. It was

designed by a Persian architect, Rafi. Its design is said to resemble the Mosque of Cordoba in

Spain. It is part of Gulbarga fort complex.

So Option (c) is correct.

ADDITIONAL INFORMATION:

Important Mosques:

Name of Mosque Location

Nagina Masjid Agra

Jama Masjid Delhi

Moti Masjid Agra

Bara Imambara Lucknow

Adina Masjid Pandua

Haji Ali Dargah Mumbai

Q.80) Match List I with List II

List I List II

A. Chandidasa 1. Radhavallabh sect

B. Chaitanya 2. Sahajiyas

Page 69: PRESTORMING 2021 SET 2 - TEST 15 HISTORY III - EXPLANATION

C. Husain Shah 3. Advaita philosophy

D. Surdasa 4. Satyapir worship

5. Bhedabheda philosophy

Codes

A B C D

(a) 2 1 4 5

(b) 2 5 4 1

(c) 1 5 3 4

(d) 3 4 2 1

EXPLANATION:

A large part of medieval Bangla literature is based on the Sahajiya creed. Among the people who

wrote on this philosophy, Baru Chandidas is considered the best. His Srikrishnakirtan depicts

the basic principles of Sahajiya in a lucid manner. Many poets including Chandidas, who

believed in the Sahajiya doctrine, composed verses on practices of the attainment of the self in an

enigmatic language, known as ragatmika pada.

Chaitanya‘s Bhakti movement was founded on the Bhedabheda philosophy of Nimbarka.

Chaitanya named his philosophy Achintya Bhedabheda. He said that as the relation of Brahman

with the world was both different and non-different, this relation could not be known or imagined

by the mind and was hence Achintya (unthinkable).

In Bengal, Husain Shah gave encouragement to the worship of Satya Pir/Satya Narayana, a

tradition where the Muslims and the Hindus shared faith in a commonly acceptable divinity with

varying names.

Surdas was the disciple of Vallabhacharya. He popularized Krishna cult in the Northern part of

India. Showed intense devotion to Radha and Krishna. He was an important proponent of the

Radhavallabh sect.

So Option (b) is correct.

ADDITIONAL INFORMATION:

Important Bhakti Saints:

Adi Shankaracharya

Ramanuja

Madhvacharya

Mirabai

Tulsidas

Ramananda

Guru Nanak

Page 70: PRESTORMING 2021 SET 2 - TEST 15 HISTORY III - EXPLANATION

Gnanadeva

Namadeva

Tukaram

Q.81) Which of the following was/were stressed by the Bhakti saints of the 15th and 16th

centuries?

1. Idea of personal God

2. Purity of caste system

3. Futility of religious rites

4. Conception of Nirguna Brahman

(a) 1 and 2

(b) 1 and 3

(c) 3 and 4

(d) 4 only

ADDITIONAL INFORMATION:

Bhakti Movement in South India:

Alwars Nayanars

They were devotees of Vishnu They were devotees of Shiva

There were 12 Alwars There were 63 Nayanars

Andal was the only female Alwar Karaikkal Ammaiyar was one of the 3

women Nayanars

Important work of them is Nalayira Divya

Prabandham

Important work of them is Tevaram/

Thirumurai

Q.82) Which of the following statements regarding the Nathpanthi-movement is/are correct?

1. It was essentially confirmed to Western India

2. It challenged the superiority of the Brahmanas

3. It advocated Hindu – Muslim unity

Select the answer from the codes given below

(a) 1 and 2

(b) 1 and 3

(c) 2 and 3

(d) 2 only

Page 71: PRESTORMING 2021 SET 2 - TEST 15 HISTORY III - EXPLANATION

EXPLANATION:

The Nathpanthi movement was at one time they very popular all over North India in areas of Punjab,

Sindh, Rajasthan, Gangetic plain, Bengal delta, etc. It was not only confined to Western India. So

Statement 1 is not correct.

Many of the yogis of this sect belonged to the lower castes. They denounced the caste system and the

privileges claimed by the Brahamans. The path they preached was open to all, irrespective of caste

distinctions. So Statement 2 is correct.

It advocated Hindu-Muslim unity. There was no distinction based on religion. It criticised the ritual and

other aspects of conventional religion and the social order. So Statement 3 is correct.

ADDITIONAL INFORMATION:

Nathpanthi Movement:

The followers of Gorakhnath were called as Nathpanthis

The path they preached was called Tantra

According to it, by uttering magical words (mantra) and making various kinds of mystic gestures,

one could attain the desires as well as gain supernatural powers

It became popular among lower castes

It was a heterodox movement

It was a Shaivist sub-tradition within Hinduism

It combined the ideas of Buddhism, Shaivism and Yoga traditions

Q.83) Amri culture flourished in

(a) The Kutch region

(b) Afghanistan

(c) Baluchistan

(d) China

EXPLANATION:

The Amri culture refers to the Amri archaeological sites in the Sindh and Baluchistan provinces of

Pakistan. It flourished in the 4th and 3rd millennia BC. It was an important site of the Harappan

Civilisation. It was located on the banks of the Indus river. There were evidences for the presence of

Rhinoceros and Antelope. This site was excavated by N G Majumdar in 1935.

So Option (c) is correct.

Page 72: PRESTORMING 2021 SET 2 - TEST 15 HISTORY III - EXPLANATION

ADDITIONAL INFORMATION:

Harappan sites in Pakistan:

Mohenjadaro

Harappa

Chanhudaro

Sutkagendor

Ganweriwala

Kot Diji

Mehrgarh

Allahdino

Balakot

Q.84) The crop which does not seem to have been known to the people of the Harappan Culture is

(a) Rice

(b) Cotton

(c) Ragi

(d) Potato

EXPLANATION:

There is archaeological evidence for the cultivation of pea, chickpea, pigeon pea, horse gram and green

gram. Several varieties of wheat have been found at Harappan sites, as well as barley of the two-rowed

and six-rowed kinds. There is evidence that the Harappans cultivated Italian millet, ragi and amaranth,

as well as sorghum and rice. The evidence for rice has come from Lothal and Rangpur in the form of

husks embedded in pottery. Cotton was another important crop. A piece of woven cotton cloth has been

found at Mohenjodaro.

ADDITIONAL INFORMATION:

Crops not known to Harappans:

Sugarcane

Groundnut

Tomatoes

Potatoes

Animals known to Harappans:

Sheep

Goat

Buffalo

Pig

Boar

Page 73: PRESTORMING 2021 SET 2 - TEST 15 HISTORY III - EXPLANATION

Deer

Gharial

Fish

Fowl

Rhinoceros

Tiger

Dog

Q.85) Which one of the following is correctly matched?

(a) Surkotada : Ploughed field

(b) Harappa : Horse remains

(c) Rangpur : Rice husk

(d) Chanhudaro: Citadel

EXPLANATION:

The remains of an animal resembling horse was found in the Harappan site of Surkotda which is

located in Gujarat

Workmen quarters was found near the granary at Harappa proved the presence of a class of

workmen. It also showed they considered the welfare of workers. Coffin burials were also found

here.

The evidence for rice in Harappan civilisation has come from Lothal and Rangpur in the form of

husks embedded in pottery

Bead making was found in the Harappan site of Chanhudaro which is located in Pakistan.

Materials for making beads included beautiful red coloured stone-like carnelian, jasper, crystal

quartz and steatite. It was the only Harappan city without a Citadel.

So Option (c) is correct.

ADDITIONAL INFORMATION:

Findings Harappan Site

Ploughed Field Kalibangan

Terracotta Plough Banawali

Dockyard Lothal

Dancing girl statue Mohenjadaro

Furrowed land Kalibangan

Page 74: PRESTORMING 2021 SET 2 - TEST 15 HISTORY III - EXPLANATION

Cemetery Harappa

Pot Burial Surkotda

Q.86) The largest number of seals of the Harappan culture are made of

(a) Terracotta

(b) Faience

(c) Agata

(d) Steatite

EXPLANATION:

Most of the seals were made of steatite, which is a kind of soft stone. A few of them were also made of

ivory, agate, chert, copper, faience and terracotta. The standard Harappan seal was square in shape with

a 2X2 square inches dimension. It is believed that the seals were used for commercial purposes. A few

seals were also carried as amulets as a kind of identity card. All the seals have pictures of animals with

something written in a pictographic script. Animals depicted on the seals were unicorn bull, rhinoceros,

tiger, elephant, bison, goat, buffalo, etc. Most of the seals have been written on both sides. The writings

are in the Kharosthi style. So Option (d) is correct.

ADDITIONAL INFORMATION:

Pashupati Seal:

It was discovered at Mohenjadaro

It was made of Steatite

This seal depicts a human figure seated cross-legged similar to yogic posture

The human figure is depicted with two horns

An elephant and a tiger are depicted to the right side of the seated figure, while on the left a

rhinoceros and a buffalo are seen

In addition to these animals, two antelopes are shown below the seat

Q.87) The famous conversation between UddalakaAruni and his son Svetaketu regarding the

identity of the brahman and the Atman figures in the

(a) Svetasvatara Upanishad

(b) Chandogya Upanishad

(c) Mundaka Upanishad

(d) Mandukya Upanishad

Page 75: PRESTORMING 2021 SET 2 - TEST 15 HISTORY III - EXPLANATION

EXPLANATION:

In the sixth chapter of Chandogya Upanishad, Uddalaka Aruni instructs his son Svetaketu in the

nature of brahman, the supreme reality. Uddalaka is portrayed as a caring father who sent his son,

at his age of twelve, to a residential school. And, on his return home, after twelve years of

education, Uddalaka questions Svetaketu, now a bright-looking well-grown young man, to find out

whether he had learned anything of importance. So Option (b) is correct.

ADDITIONAL INFORMATION:

Important Upanishads:

Aitareya Upanishad: Talks about the creation of Atman (Soul) and Consciousness

Brihadaranyaka Upanishad: Talks about the transmigration of Atman; passages on metaphysics

and ethics

Chandogya Upanishad: Deals with rhythm and chanting of Mantras

Katho Upanishad: Tells the story of Nachiketa and Yama

Mundaka Upanishad: Contains the mantra ―Satyameva Jayate‖ (truth alone triumphs) which is

borrowed in the National Emblem of India

Svetasvatara Upanishad: In this Upanishad Siva or Rudra is declared to be the creator, preserver

and destroyer of the world. He is the material and efficient cause of this world. He is identified with

the Supreme Brahman

Q.88) What is the correct sequence of the following in the history of South India?

1. Expansion of the Mauryan rule

2. Beginnings of the megalithic culture

3. Sangam Age

4. Pallava ascendancy

(a) 2,3,1,4

(b) 2,1,3,4

(c) 2,1,4,3

(d) 1,2,3,4

EXPLANATION:

According to Sangam Literature Maurya invaded up to the far south. Therefore it can be said that

during the rule of Bindusara, the Mauryan dynasty extended as far as Mysore and therefore

included almost the whole of India but excluded a small portion of the unexplored tribal and

forested regions near Kalinga (Odisha) and the kingdoms of extreme south were not the part of

the empire. The Kalinga was also later annexed by Bindusara‘s son Ashoka. These events

happened in the period of the 3rd century BC.

Page 76: PRESTORMING 2021 SET 2 - TEST 15 HISTORY III - EXPLANATION

The beginning of the Megalithic culture in South India dates back to 1000 BC. The megalithic

culture is known for its burials. This period corresponds to the Iron Age culture.

The period between 300 BC to 300 AD is considered as the Sangam Age of South India. It was in

this period that the three Sangams or Academy of Poets was held in South India. The Sangam

Age corresponds to the rule of South India by the famous Cheras, Cholas and Pandyas.

The ascendancy of the Pallava rule in South India began in the 6th century AD. Pallavas became

dominant in the 6th century after a successful attack against the Kalabhras, which extended their

territory as far south as the Kaveri River. The Pallavas reached their zenith during the reign of

Mahendravarman I (600–630 AD), a contemporary of Harsha and Pulakeshin II.

Hence the correct sequence of events is 2-1-3-4. So Option (b) is correct.

ADDITIONAL INFORMATION:

Important Megalithic Sites in South India:

Adichanallur – Tamil Nadu

Payampalli - Tamil Nadu

Kodumanal - Tamil Nadu

Hallur - Karnataka

Kumathi – Karnataka

Brahmagiri - Karnataka

Naikund - Vidarbha

Takalghat – Vidarbha

Ariyanur - Kerala

Cheramanangad – Kerala

Q.89) Which of the following believed in the workship of the Yakshas and Yakshinis?

1. Brahmanism

2. The Kalamukha sect

3. Buddhism

4. Jainism

(a) 1, 2 and 3

(b) 1, 2 and 4

(c) 1, 3 and 4

(d) 2, 3 and 4

EXPLANATION:

Yakshas and Yakshinis as art in the form of sculptures began in the Mauryan period. Yaksha is the

male deity and Yakshini is the female counterpart. They are a class of spirit beings or semi-divine

Page 77: PRESTORMING 2021 SET 2 - TEST 15 HISTORY III - EXPLANATION

beings who are mentioned in Hindu, Buddhist and Jain literature as inhabitants of the subterranean

earth and protectors of treasures. In Hindu literature, they are described as the giver of wealth, lord

of wealth, king of kings and king of hidden mysteries. In the Buddhist and Jain art and sculpture

they are depicted as associates and attendant deities of Bodhisattvas and Tirthankaras. Yaksha

worship was very popular before and after the advent of Buddhism and it was well assimilated in

Buddhism and Jainism.

Kalamukha Sect is a sect that worshipped the god Shiva as the supreme deity. It was prominent in

India from the 8th to the 13th century and not associated with Yakshas and Yakshinis.

So Option (c) is correct.

ADDITIONAL INFORMATION:

Kalamukha Sect:

It is a sect that worshipped the god Shiva as the supreme deity

It was prominent in India from the 8th to the 13th century

It was considered as one of the successors of Pashupata sect

Another Shaivite sect associated with it is the Kaplika sect

The Kalamukhas held that happiness in this world and salvation in the next could be attained by

such practices as:

Eating food in a human skull

Besmearing the body with the ashes of the dead and also eating those ashes

Worshipping the God as seated in a pot of wine and

Holding a club

Q.90) The doctrine that the human personality consists of five skandhas is associated with

(a) Buddha

(b) Mahavira

(c) Parsvanatha

(d) MaskariputraGosala

EXPLANATION:

The Five Skandhas are the five elements that sum up the whole of an individual‘s mental and physical

existence. The Buddha spoke often of these Five Skandhas, also called the Five Aggregates or the Five

Heaps. The self/soul cannot be identified with any one of the parts, nor is it the total of the parts. The

skandhas, very roughly, might be thought of as components that come together to make an

individual.So Option (a) is correct.

Page 78: PRESTORMING 2021 SET 2 - TEST 15 HISTORY III - EXPLANATION

ADDITIONAL INFORMATION:

Five Skandhas:

1. Rupa : Form, matter or body

2. Vedana : Sensation or feelings

3. Samjna : Perceptions of sense objects

4. Samskara : Mental formations

5. Vijnana : Awareness or consciousness

Q.91) The concept of the Eight-fold forms the theme of

(a) Dharma Chakra PravartanaSutta

(b) Divyavadana

(c) Dipavamsa

(d) MahaparinibbanSutta

EXPLANATION:

In Buddhist symbolism, the Noble Eightfold Path is often represented by means of the Dharma

wheel/Dharma Chakra, whose eight spokes represent the eight elements of the path. The Noble Eightfold

Path is one of the principal teachings of the Buddha, who described it as the way leading to the

cessation of Dukkha or suffering and attainment of self-awakening. The Eightfold Path includes right

understanding, right speech, right livelihood; right mindfulness, right thought, right action, right effort

and right concentration.

So Option (a) is correct.

ADDITIONAL INFORMATION:

Divyavadana:

It means divine tales

It is an anthology of Buddhist narratives

It is an anthology of 36 Buddhist avadanas/narratives and two sutras that celebrate the lives of

exemplary personalities in the history of Buddhism

It was written in Sanskrit

These narratives revolve around the Buddha explaining to his disciples the functioning of

retributive karma

Dipavamsa:

It means Chronicle of Island

It is the oldest historical record of Sri Lanka

It was compiled in the period of 3rd or 4th century BC

Page 79: PRESTORMING 2021 SET 2 - TEST 15 HISTORY III - EXPLANATION

It was written in Pali

It gives historical and mythical accounts of the Buddha‘s life, Buddhist Councils, Ashoka and the

arrival of Buddhism to Sri Lanka

Mahaparinibbana Sutta:

It was written in Pali

It describes the Buddha‘s last days, his passage into nirvana, his funeral, and the distribution of

his relics

According to this text, the death of Lord Buddha at the age of 80 is considered as the original

Mahaparinirvana

Mahaparinirvana is one of the major goals of Buddhism which means release or freedom after

death

Q.92) Nagarjuna‟sSunyaveda is expounded in

(a) Yogachara

(b) Vaibhashika

(c) Madhyamika

(d) Sautrantika

EXPLANATION:

Sunyavada is a Buddhist philosophy. It was developed by 2nd-century Indian philosopher Nagarjuna. It

emerged out of the Madhyamika school and sometimes treated synonymous to it. It states that the

voidness constitutes ultimate reality. It is seen not as a negation of existence but rather as the

undifferentiation out of which all apparent entities, distinctions, and dualities arise. Hence Nagarjuna‘s

Sunyavada is expounded in Madhyamika. So Option (c) is correct.

Yogachara:

It is an influential idealistic school of Mahayana Buddhism

Yogachara school developed during the fourth century during Gupta and Pala period

It emphasizes the study of cognition, perception, and consciousness through the interior lens of

meditative and yogic practices

Asanga and Vasubandhu were two philosophers of the Yogachara school

Vaibhashika:

It belongs to Hinayana Buddhism

It was derived from the Sravastivadin school of realism

It is believed to be formed around the 3rd century BC

It was found chiefly in Gandhara and Kashmir

It expounds the philosophy of Sarvam Asti or everything exists

Page 80: PRESTORMING 2021 SET 2 - TEST 15 HISTORY III - EXPLANATION

Sautrantika:

It is an ancient school of Buddhism

It emerged in India about the 2nd century BC as an offshoot of the Sarvastivada school of realism

It rejects the authority of the Abhidharma

Sometimes it is characterized as a transitional school that led to the development of the Mahayana

tradition, and many of its views influenced later Yogachara thought

Q.93) Biographies of JainaTirthankaras are found in

(a) Bhagavati sutra

(b) Kalpa sutra

(c) Niryavali sutra

(d) Uvasagadasao

EXPLANATION:

Kalpa Sutra is Jain literature written by Bhadrabahu who was one of the major followers of the

Digambara sect in Jainism. It contains the biographies of the 24 Tirthankaras. It mainly talks about

Pashavanatha, the last Tirthankara prior to Lord Mahavira, and Mahavira himself. It also mentions that

Rishabhanatha was the 1st Tirthankara. So Option (b) is correct.

ADDITIONAL INFORMATION:

Bhagavati sutra:

It is the 5th in the 12 Jain Angas

It is also called as Vyakhyaprajnapti

It was written in Prakrit

It was composed by Sudharmaswami

It contains details about questions answered by Mahavira

Niryavali sutra:

It is a Jain text

It is also called as Panchopang Satika

It contains commentaries of Shreechandra Suri

It is written in Prakrit and Sanskrit

Uvasagadasao:

It is also called as Uvasaggaharam Stotra

It is a Jain text

It was written by Bhadrabahu

This text speaks about Prasvanatha , who was the 23rd Jain Tirthankara

Page 81: PRESTORMING 2021 SET 2 - TEST 15 HISTORY III - EXPLANATION

Q.94) Which of the following were meant by Ashoka by the term dhamma?

1. Firm attachment to only one‘s own reigion

2. Taking care of mother and father

3. Non-violence to living beings

4. Moderation in both spending and saving

Select the correct answer from the codes given

(a) 1 only

(b) 1 and 3

(c) 2, 3 and 4

(d) 1, 2, 3 and 4

EXPLANATION:

Ashoka‘s dhamma included religious tolerance which involved the acceptance of all religions and

providing equal treatment to all. Hence it did not advocate firm attachment to only one‘s own

religion. So, statement 1 is not correct.

Ashoka said everybody should serve parents, revere teachers. He said respecting and taking care

of mother and father was the prime duty of their children.

He promoted non-violence to living beings. He asked everyone to avoid animal slaughter and

sacrifice. He also expounded humane treatment of animals, servants and prisoners.

According to Ashoka‘s dhamma moderation in spending and moderation in saving is good. It was

conveyed through the rock inscription at Girnar.

Hence only 2, 3 and 4 are correct. So Option (c) is correct.

ADDITIONAL INFORMATION:

Ashoka‟s Dhamma:

Ashoka appointed officials, known as the dhamma mahamatta who went from place to place

teaching people about dhamma

He got his messages inscribed on rocks and pillars, instructing his officials to read his message to

those who could not read it themselves

In most of these inscriptions Ashoka was referred to as Devanampiya Piyadasi

Most of his edicts are written in Pali and Prakrit in Brahmi script. Some are written in the

Kharoshti , Aramaic and Greek as well.

Ashoka also sent messengers to spread ideas about dhamma to other lands, such as Syria, Egypt,

Greece and Sri Lanka

He built roads, dug wells, and built rest houses

He arranged for medical treatment for both human beings and animals

Page 82: PRESTORMING 2021 SET 2 - TEST 15 HISTORY III - EXPLANATION

Q.95) The official credited with the construction of the sudarshana lake during the reign of

Chandragupta Maurya was

(a) YavanaragaTusaspha

(b) Parnadatta

(c) PahlavaSuvisakha

(d) Vaisya Pusyagupta

EXPLANATION:

The Sudarshana lake is located in Girnar in Saurashtra. Evidence regarding the lake is mentioned in the

Junagadh rock inscription of Rudradaman. It mentions that this lake was built by one of Chandragupta

Maurya‘s governor, Vaisya Pushyagupta. The Sanskrit edict also notes the repairs done by Mahakshtrap

Rudradaman on Sudarshana Lake. It mentions a Yavana king named Tushaspha building conduits here

during the reign of Ashoka. The Junagadh inscription of Skandagupta states that Sudarshana Lake was

repaired during the rule of Skandagupta by his governor Parnadatta.

So Option (d) is correct.

ADDITIONAL INFORMATION:

Chandragupta Maurya:

He was the founder of the Mauryan empire

He overthrew the Nanda dynasty with the help of Chanakya also called as Kautilya

Chandragupta Maurya ruled from 324 BC to 297 BC

Pataliputra served as the capital of his empire

He became a disciple of Bhadrabahu, a Jain saint and embraced Jainism

Literary evidences about his rule:

Arthashastra by Kautilya

Megasthenes‘ Indica

Devichandraguptam and Mudrarakshasa by Vishakhadatta

He was succeeded by his son Bindusara

Q.96) Match List I with List II and select the correct answer

List I List II

A. Pratiharas 1. Suvamagiri

B. Chaulukyas 2. Chandravati

C. Chahamanas 3. Anahilapataka

D. Paramaras 4. Kanyakubja

5. Kalyani

Codes

Page 83: PRESTORMING 2021 SET 2 - TEST 15 HISTORY III - EXPLANATION

A B C D

(a) 2 3 4 1

(b) 4 3 1 2

(c) 1 2 4 5

(d) 3 5 2 4

EXPLANATION:

Kanyakubja is the modern-day Kanauj located in Uttar Pradesh. It remained a focal point for the

struggle between the Palas, Pratiharas and Rashtrakutas. This was called the Tripartite struggle.

The greatest ruler of the Pratihara dynasty was Mihir Bhoja. He recovered Kanauj/Kanyakubja by

836 AD, and it remained the capital of the Pratiharas for almost a century.

Mulraja I was an important ruler of the Chalukyas of Gujarat. He defeated the last Chanda king

of Gujarat and founded an Independent kingdom with his capital in Anahilapataka in 940 AD.

Chahamanas were also called as Chauhans of Shakambari. They ruled the region with

Shakambari as their capital in the 11th century. It is located in Rajasthan.

Chandravati is located on the banks of the West Banas River in the state of Rajasthan. It was

ruled by the Paramaras of Abu. It served as their capital during their period of rule between the

10th and 13th centuries.

Hence the correct code is A-4, B-3, C-1, D-2. So Option (b) is correct.

ADDITIONAL INFORMATION:

Dynasty Capital

Mauryas Pataliputra

Kushanas Purushpura/Peshawar

Satavahanas Amaravathi

Chera Vanji

Chola Puhar, Thanjavur

Pandyas Madurai

Pallavas Kanchi

Chalukyas Badami

Rashtrakutas Manyakheta

Page 84: PRESTORMING 2021 SET 2 - TEST 15 HISTORY III - EXPLANATION

Q.97) Which one of the following regions is known for paintings that are manuscript illustrations

in miniature executed on palm leaf during the period A.D. 1100-1300?

(a) Kerala

(b) Deccan

(c) The Chola Kingdom

(d) Western India

EXPLANATION:

In Western India between the 10th to 12th-century miniature painting developed. This new style figured

first in the form of illustrations etched on palm-leaf manuscripts. The contents of these manuscripts

included literature on Buddhism and Jainism. It was in the 14th century, that paper replaced the palm

leaf.

Hence Western India is known for paintings that are manuscript illustrations in miniature executed on

palm leaf during the period A.D. 1100-1300. So Option (d) is correct.

ADDITIONAL INFORMATION:

Different Schools of Miniature painting:

Pala School

Western Indian School

Mughal School

Deccani Schools:

Ahmednagar

Bijapur

Golconda

Hyderabad

Tanjore

Central Indian and Rajasthani Schools

Pahari Schools

Q.98) Somapura, the site of a great Buddhist monastery was located in

(a) Eastern Bihar

(b) Northern Bihar

(c) Northern Bengal

(d) Southern Bengal

Page 85: PRESTORMING 2021 SET 2 - TEST 15 HISTORY III - EXPLANATION

EXPLANATION:

Somapura vihara/monastery was established in North Bengal at the end of the 8th or the beginning of

the 9th century AD by Dharmapala. It is one of the biggest single-unit Buddhist monasteries in South

Asia, organised around a square courtyard with an entrance from the north. In its centre, the imposing

brick shrine is a conspicuous height of over 21 metres, and it is decorated with friezes of stone

sculptures and terracotta plaques.

So Option (c) is correct.

ADDITIONAL INFORMATION:

Famous Buddhist monasteries in India:

Name of Monastery Location

Tawang Monastery Arunachal Pradesh

Ghum Monastery West Bengal

Thiksey Monastery Ladakh

Hemis Monastery Ladakh

Tsulglagkhang Monastery Himachal Pradesh

Kye Gompa Monastery Himachal Pradesh

Namdroling Nyingmapa Monastery Karnataka

Rumtek Monastery Sikkim

Q.99) Match List I with List II and select the correct answer

List I

( Years)

List II

( Events)

A. 1296 A.D. 1. The Mongols Invaded Delhi

B. 1336 A.D. 2. Harihara founded the kingdom of

Vijayanagara

C. 1398 A.D. 3. Timur‘s invasion

D. 1504 A.D. 4. The city of Agra founded by Sultan Sikandar

Lodi

5. Transfer of the capital to Daulatabad from

Delhi

Page 86: PRESTORMING 2021 SET 2 - TEST 15 HISTORY III - EXPLANATION

Codes

A B C D

(a) 2 3 4 5

(b) 1 2 3 5

(c) 3 1 5 4

(d) 1 2 3 4

EXPLANATION:

The Mongols of the Chaghatai Khanate under Duwa Khan tried to invade Delhi in 1296 AD.

Alauddin Khilji sent an army commanded by his brother Ulugh Khan and the general Zafar

Khan, and this army comprehensively defeated the Mongols, with the capture of 20,000

prisoners, who were put to death.

In 1336 AD, the kingdom of Vijayanagara was founded by Harihara and Bukka in the wake of the

rebellions against Tughluq rule in the Deccan. They both belonged to the Sangama dynasty. It

was located on the southern banks of the river Tungabhadra.

It was during the reign of the last king of the Tughlaq dynasty that the mighty king Timur or

Tamerlane invaded India in 1398 A.D. He crossed Indus and captured Multan, and just walked

over to Delhi without much resistance.

In 1504 AD the city of Agra was founded by Sultan Sikandar Lodi. Later he also made Agra the

capital of Delhi Sultanate. It served as a base from which it would enable him to launch his

attempt to control Malwa and Rajasthan.

Hence the correct code is A-1, B-2, C-3, D-4. So option (d) is correct.

ADDITIONAL INFORMATION:

Transfer of the capital to Daulatabad from Delhi:

It was done by Muhammad-bin-Tughlaq

It happened in the year 1327 AD

Reasons for transfer of capital:

Daulatabad was centrally located and equidistant from Delhi and other important places

Delhi was within the reach of the Mongols and Daulatabad appeared to be at a safe

distance from the possible Mongol attacks in the future

Q.100) The term pictra dura refers to

(a) Wall painting

(b) Inlay work

Page 87: PRESTORMING 2021 SET 2 - TEST 15 HISTORY III - EXPLANATION

(c) Sculptures in Italian style

(d) A special type of Turrets

EXPLANATION:

Pietra dura also called as parchin kari, is a term for the inlay technique of using cut and fitted, highly

polished coloured stones to create images. The works in pietra dura often represent floral motifs, plants,

or natural settings. This technique was first used in the 16th century in Rome and later developed in

Florence, Italy. It is employed in Taj Mahal built by Shah Jahan.

Hence the term pietra dura refers to inlay work. So Option (b) is correct.

ADDITIONAL INFORMATION:

Arabesque:

It is surface decorations based on rhythmic linear patterns of scrolling and interlacing foliage,

tendrils

It is an essential part of the decorative tradition of Islamic cultures and employed in their art and

architecture

These designs usually were composed of either twining or sinuous scrolls of branches and leaves or

ornate lines abstracted from such natural forms

Human figures often were integral to Western arabesque designs